Sei sulla pagina 1di 106

ADailyInitiativefromCivilsdaily|DownloadFREEApp|JointhePrelimsModule

PRELIMSDAILY
8Questions+1Tikdam/Titbitsinexplanations

----------------------------------------------------------------

1June2017QuestionsandAnswers

Q.1) Which of the following organizations is a partner in the Global Nutrition Report?

a) Generations United

b) CARE

c) World Health Organisation

d) Food and Agriculture Organisation

Answer: (c)

The Global Nutrition Report acts as a report card on the worlds nutritionglobally, regionally, and
country by countryand on efforts to improve it. It assesses progress in meeting Global Nutrition Targets
established by the World Health Assembly. The World Health Organization is a Global Nutrition Report
Partner. The Global Nutrition Report convenes existing processes, highlights progress in combating
malnutrition and identifies gaps and proposes ways to fill them. Through this, the Report helps to guide
action, build accountability and spark increased commitment for further progress towards reducing
malnutrition much faster.

Q.2) Which of the following statement given below is incorrect regarding the Bioremediation?

a) Bioremediation is a waste management technique that involves the use of organisms to neutralize
pollutants from a contaminated site

b) Incineration is a type of Bioremediation.

c) Bioremediation Technology can be generally classified as in situ or ex situ.

d) All are correct

PrelimsDailyArchive-Clickhere|TargetMainsArchive-Clickhere

ADailyInitiativefromCivilsdaily|DownloadFREEApp|JointhePrelimsModule

Answer: (b)

Incineration: A waste treatment technology, which includes the combustion of waste for recovering
energy, is called as incineration. Incineration coupled with high temperature waste treatments are
recognized as thermal treatments. During the process of incineration, the waste material that is treated is
converted in to IBM, gases, particles and heat. These products are later used for generation of electricity.
The gases, flue gases are first treated for eradication of pollutants before going in to atmosphere.

Bioremediation is a waste management technique that involves the use of organisms to neutralize
pollutants from a contaminated site. Bioremediation is a treatment that uses naturally occurring
organisms to break down hazardous substances into less toxic or non toxic substances

Q.3) NISARGRUNA technology is related to

a) Biomethanation

b) Incineration

c) Bioremediation

d) None of the above

Answer: (a)

NISARGRUNA technology for biomethanation was developed by Sharad Kale, a professor at the Bhabha
Atomic Research Centre at Mumbai

Q.4) Consider the following statements:

1. Geocoding can help users find a wide variety of location-specific information from a device.

2. Geotagging refers to the process of taking non-coordinate based geographical identifiers, such as a
street address, and finding associated geographic coordinates.

Which of the statements is/are correct?

a) 1 only

b) 2 only

c) 1 and 2 both

d) Neither 1 nor 2

Answer: (d)

PrelimsDailyArchive-Clickhere|TargetMainsArchive-Clickhere

ADailyInitiativefromCivilsdaily|DownloadFREEApp|JointhePrelimsModule

Geotagging can help users find a wide variety of location-specific information from a device. Geocoding
refers to the process of taking non-coordinate based geographical identifiers, such as a street address,
and finding associated geographic coordinates.

Q.5) If money bill is defeated in the Lok Sabha, what will happen?

a) The minister who introduced the Bill must resign.

b) It leads to the resignation of the Government.

c) The private member who introduced the bill must resign.

d) President will dissolve Lok Sabha

Answer: (b)

If money bill is defeated in the Lok Sabha, it leads to the resignation of the Government. Money bill
cannot be introduced by a private member. If money bill is defeated in the Lok Sabha, President of India
has no power to dissolve Lok Sabha.

Q.6) With reference to food chains in ecosystems, consider the following statements :

1. A food chain illustrates the order in which a chain of organisms feed upon each other.

2. Food chains are found within the populations of a species.

3. A food chain illustrates the numbers of each organism which are eaten by others.

Which of the statements given above is / are correct?

a) 1 only

b) 1 and 2 only

c) 1, 2 and 3

d) None

Answer: (a)

A food chain is the sequence of who eats whom in a biological community (an ecosystem) to obtain
nutrition.

Q.7) Consider the following pairs

National Park River flowing through the Park

PrelimsDailyArchive-Clickhere|TargetMainsArchive-Clickhere

ADailyInitiativefromCivilsdaily|DownloadFREEApp|JointhePrelimsModule

1. Corbett National Park : Ganga

2. Kaziranga National Park : Manas

3. Silent Valley National Park : Kaveri

Which of the above pairs is/are correctly matched?

a) 1 and 2

b) 3 only

c) 1 and 3

d) None

Answer: (d)

Ramganga, a tributary of Ganga passes through Corbett NP but Ganga does. Kaveri and Manas also do
not pass through Silent valley and Kaziranga NP respectively.

Q.8) Consider the following organisms

1. Agaricus

2. Nostoc

3. Spirogyra

Which of the above is / are used as biofertilizer / biofertilizers

a) 1 and 2

b) 2 only

c) 2 and 3

d) 3 only

Answer: (b)

Agaricus:-Agaricus is a genus of mushrooms containing both edible and poisonous species, with possibly
over 300 members worldwide.Nostoc:-Bioinoculants are the microbial preparations that enhance the
uptake of nutrients by plants from the soil, increase the avaliability of the nutrients and stimulate plant
growth.There are two types of organisms which are used as inoculants: symbiotic organisms (such as
Rhizobium, Synorhizobium, Bradyrhizobium, Nostoc, Anabaena, etc.) and non-symbiotic organisms (such

PrelimsDailyArchive-Clickhere|TargetMainsArchive-Clickhere

ADailyInitiativefromCivilsdaily|DownloadFREEApp|JointhePrelimsModule

as Azotobacter, Azospirilum, Beijerinckia, etc.)Spirogyra:-Spirogyra is a genus of filamentous green algae
of the order Zygnematales, named for the helical or spiral arrangement of the chloroplasts that is
diagnostic of the genus.

PrelimsDailyArchive-Clickhere|TargetMainsArchive-Clickhere

ADailyInitiativefromCivilsdaily|DownloadFREEApp|JointhePrelimsModule

PRELIMSDAILY
8Questions+1Tikdam/Titbitsinexplanations

----------------------------------------------------------------

02June2017QuestionsandAnswers

Q.1) Which of the following statements regarding the Preventive Detention is incprrect?

a) If a person is arrested or detained under a law providing for preventive detention, then the protection
against arrest and detention under Article 22 (1) and 22 (2) shall not be available

b) A detainee under preventive detention can have no right of personal liberty guaranteed by Article 19 or
Article 21

c) A person may be taken to preventive custody only for 3 months at the first instance

d) All are correct

Answer: (d)

The Article 22 (3) of the Indian constitution provides that, if a person is arrested or detained under a law
providing for preventive detention, then the protection against arrest and detention under Article 22 (1)
and 22 (2) shall not be available.Preventive detention on the other hand is action taken beforehand to
prevent possible commitment of crime. Preventive detention thus is action taken on grounds of suspicion
that some wrong actions may be done by the person concerned

Q.2) Consider the following statments regarding the National Food Security Act(NFSA), 2013:

1. The NFSA 2013 recognizes maternity entitlements

2. Under NFSA, States have the responsibility of identifying the eligible households.

Which of the statements given above is/are correct?

a) Both are correct

PrelimsDailyArchive-Clickhere|TargetMainsArchive-Clickhere

ADailyInitiativefromCivilsdaily|DownloadFREEApp|JointhePrelimsModule

b)1 only

c) 2 only

d) None is correct

Answer: (a)

The National Food Security Act, 2013 (also Right to Food Act) is an Act of the Parliament of India which
aims to provide subsidized food grains to approximately two thirds of India's 1.2 billion people. It was
signed into law on 12 September 2013, retroactive to 5 July 2013. The National Food Security Act, 2013
(NFSA 2013) converts into legal entitlements for existing food security programmes of the Government of
India. It includes the Midday Meal Scheme, Integrated Child Development Services scheme and the
Public Distribution System. Further, the NFSA 2013 recognizes maternity entitlements. The Midday Meal
Scheme and the Integrated Child Development Services Scheme are universal in nature whereas the
PDS will reach about two-thirds of the population (75% in rural areas and 50% in urban areas).

Q.3) Food Price Index is released by:

a) UNDP

b) FAO

c) Trifed

d) Amnesty International

Answer: (b)

The FAO food price index is a measure of the monthly change in international prices of a market basket
of food commodities. It consists of the average of five commodity group price indices, weighted with the
average export shares of each of the groups for 2002-2004: FAO Cereal price index.

PrelimsDailyArchive-Clickhere|TargetMainsArchive-Clickhere

ADailyInitiativefromCivilsdaily|DownloadFREEApp|JointhePrelimsModule

Q.4) Consider the following statements:

1. Guar or cluster bean is used as a thickening agent in the fracking fluid

2. In India, Rajasthan, Gujarat and Haryana are the main producing regions of Guar

Which of the above is/are correct?

a) 1 only

b) 2 only

c) Both 1 and 2

d) Neither 1 nor 2

Answer: (c)

Guar gum, also called guaran, is a substance made from guar beans which has thickening and stabilising
properties useful in various industries, traditionally the food industry, but increasingly the hydraulic
fracturing industry. In India, Rajasthan, Gujarat and Haryana are the main producing regions, and
Jodhpur, Sri Ganganagar and Hanumangarh in Rajasthan are the major Guar trading markets.

Q.5) Arrange the following places from north to south direction.

1. Tashkent

2. Ashgabat

3. Astana

4. Kabul

Select the correct answer using the codes given below:

a) 1-3-2-4

b) 3-1-2-4

c) 3-2-1-4

d) 4-2-1-3

Answer: (b)

PrelimsDailyArchive-Clickhere|TargetMainsArchive-Clickhere

ADailyInitiativefromCivilsdaily|DownloadFREEApp|JointhePrelimsModule

Q.6) Which of the following adds / add nitrogen to the soil?

1. Excretion of urea by animals

2. Burning of coal by man

3. Death of vegetation

Select the correct answer using the codes given below.

a) 1 only

b) 2 and 3 only

c) 1 and 3 only

d) 1, 2 and 3

Answer: (c)

Carnivorous animals feed on herbivorous animals that live on plants. When animals defecate, this waste
material is broken down by worms and insects mostly beetles and ants. These small soil animals break
the waste material into smaller bits on which microscopic bacteria and fungi can act. This material is thus
broken downfurther into nutrients that plants can absorb and use for their growth. Thus nutrie nts are
recycled back from animals to plants. Similarly the bodies of dead animals are also broken down into
nutrients that are used by the plants for their growth. Thus the nitrogen cycle on which life is dependent is
completed. Statement 2 is wrong as burning of coal does not produce Nitrogen of its compounds.

Q.7) In which of the following States is liontailed macaque found in its natural habitat?

1. Tamil Nadu

2. Kerala

3. Karnataka

4. Andhra Pradesh

Select the correct answer using the codes given below.

a) 1, 2 and 3 only

b) 2 only

PrelimsDailyArchive-Clickhere|TargetMainsArchive-Clickhere

ADailyInitiativefromCivilsdaily|DownloadFREEApp|JointhePrelimsModule

c) 1, 3 and 4 only

d) 1, 2, 3 and 4

Answer: (a)

The lion- tailed macaque is one of the most endangered primates of India , confined in distribution to the
rainforests of southern Western Ghats. Habitat specialists , they have adapted themselves to the
evergreen environments available in the south Indian states of Kerala, Karnataka and Tamil Nadu.

Q.8) Some Buddhist rockcut caves are called Chaityas, while the others are called Viharas. What is the
difference between the two?

a) Vihara is a place of worship, while Chaitya is the dwelling place of the monks

b) Chaitya is a place of worship, while Vihara is the dwelling place of the monks

c) Chaitya is the stupa at the far end of the cave, while Vihara is the hall axial to it

d) There is no material difference between the two

Answer: (b)

Chaityas refer to the halls enclosing the stupas. Chaityas were probably constructed to hold large
numbers of devotees for prayer. Viharas on the other hand are constructions built in ancient India in order
to provide resting places for the wandering Buddhist monks.

PrelimsDailyArchive-Clickhere|TargetMainsArchive-Clickhere

ADailyInitiativefromCivilsdaily|DownloadFREEApp|JointhePrelimsModule

PRELIMSDAILY
8Questions+1Tikdam/Titbitsinexplanations

----------------------------------------------------------------

3June2017QuestionsandAnswers

Q.1) The Term 'Intifada' is related to

a) Spain

b) Israel

c) Russia

d) Monglolia

Answer: (b)

Intifada is the Palestinian uprising against Israeli occupation of the West Bank and Gaza Strip. The first
intifada lasted from 1987 to 1993, and the second began in 2000.

Q.2) Consider the following statements regarding the Paris agreement and the Kyoto Protocol:

1. The Paris Agreement will come into effect after 2020.

2. The Kyoto Protocol is linked to the United Nations Framework Convention on Climate Change.

Which of the following statements given above is/are correct?

a) Both are correct

b) None is correct

c) 2 only

d) 1 only

Answer: (a)

PrelimsDailyArchive-Clickhere|TargetMainsArchive-Clickhere

ADailyInitiativefromCivilsdaily|DownloadFREEApp|JointhePrelimsModule

The agreement in Paris will come into effect only after 2020 when the Kyoto Protocol, an existing
international mechanism to deal with climate change, comes to an end.

Q.3) Consider the following statements with reference to Bhakti Movements:

1. The Bhakti movement originated in 7th century in Karnataka

2. The movement in South India favoured devotion to Krishna, Vishnu and his avatars

3. The movement in north India was centered to Rama and Shiva

Which of the statements are correct?

a) 1 and 2 only

b) 2 and 3 only

c) 1 only

d) None of the above

Answer: (d)

The Bhakti movement originated in 7th century in Tamil Nadu. The movement in South India favoured
devotion to Shiva, Vishnu and his avatars. The Movement in north India was centered to Rama and
Krishna.

Q,4) Consider the following:

1. China

2. Kazakhstan

3. Kyrgyzstan

4. Tajikistan

5. Pakistan

Which of the above is/are members of SCO?

a) 1 and 2 only

b) 1, 3 and 4

c) 2, 3 and 5

PrelimsDailyArchive-Clickhere|TargetMainsArchive-Clickhere

ADailyInitiativefromCivilsdaily|DownloadFREEApp|JointhePrelimsModule

d) All of the above

Answer: (d)

China, Kazakhstan, Kyrgyzstan, Russia, Tajikistan, and Uzbekistan are present members of SCO. India
and Pakistan will officially join as permanent members on 9 June 2017. India and Pakistan signed the
memorandum of obligations on 24 June 2016 at Tashkent, thereby starting the formal process of joining
the SCO as full members.

Q,5) Consider the following statements:

1. INDRA is a joint, bi-annual military exercise conducted by India and Russia tasked with boosting
cooperation and interoperability between the Russian and Indian navies.

2. Indra-2017 will be tri-Services exercises and will be held first time between both partners.

Which of the above is/are correct?

a) 1 only

b) 2 only

c) Both 1 and 2

d) Neither 1 nor 2

Answer: (c)

INDRA is a joint, bi-annual military exercise conducted by India and Russia starting in 2003. The exercise
is tasked with boosting cooperation and interoperability between the Russian and Indian navies. The word
INDRA is a portmanteau of the participants' respective countries. The two countries have decided to hold
the first tri-Services exercises, named Indra-2017, this year.

Q.6) Which one of the following describes best the concept of Nirvana in Buddhism?

a) The extinction of the flame of desire

b) The complete annihilation of self

c) A state of bliss and rest

d) A mental stage beyond all comprehension

Answer: (a)

PrelimsDailyArchive-Clickhere|TargetMainsArchive-Clickhere

ADailyInitiativefromCivilsdaily|DownloadFREEApp|JointhePrelimsModule

Buddha did not define Nirvana, as a state possessing few characteristics. Buddhism defines Nirvana as
being free from desire, pain, pleasure etc. It is like an extinction of flame

Q.7) According to the Constitution of India, which of the following are fundamental for the governance of
the country?

a) Fundamental Rights

b) Fundamental Duties

c) Directive Principles of State Policy

d) Fundamental Rights and Fundamental Duties

Answer: (c)

Article 37 says that Directive Principles are fundamental for the governance of country

Q.8) The people of India agitated against the arrival of Simon Commission because

a) Indians never wanted the review of the working of the Act of 1919

b) Simon Commission recommended the abolition of Dyarchy (Diarchy) in the Provinces

c) there was no Indian member in the Simon Commission

d) the Simon Commission suggested the partition of the country

Answer: (c)

In November 1927, the British govt. appointed the Indian statutory commission, known popularly after the
name of its chairman as the Simon Commission. All the members of the commission were Englishmen.
This announcement was greeted with chorus of protest from all Indians. What angered them most was
the exclusion of Indians from the commission and the basic notion behind this exclusion was that
foreigners would discuss and decide upon India's fitness for Self- government.

PrelimsDailyArchive-Clickhere|TargetMainsArchive-Clickhere

ADailyInitiativefromCivilsdaily|DownloadFREEApp|JointhePrelimsModule

PRELIMSDAILY
8Questions+1Tikdam/Titbitsinexplanations

----------------------------------------------------------------

5June2017QuestionsandAnswers

Q.1) Which of the following is not an Artillery Gun used by Indian Army?

a) Indian Field Gun

b) M-46

c) INSAS

d) All of the above

Answer: (c)

INSAN is a small arm system of Indian Army and not an artillery gun.

Q.2) Consider the following statements regarding the '2015 United Nations Climate Change Conference':

1. It was the 21st yearly session of the Conference of the Parties (COP)

2. The conference negotiated the Paris Agreement

Which of the following statements given above is/are correct?

a) 1 only

b) Both are correct

c) None is correct

d) 2 only

Answer: (b)

PrelimsDailyArchive-Clickhere|TargetMainsArchive-Clickhere

ADailyInitiativefromCivilsdaily|DownloadFREEApp|JointhePrelimsModule

The 2015 United Nations Climate Change Conference, COP 21 or CMP 11 was held in Paris, France,
from 30 November to 12 December 2015. It was the 21st yearly session of the Conference of the Parties
(COP) to the 1992 United Nations Framework Convention on Climate Change (UNFCCC) and the 11th
session of the Conference of the Parties (CMP) to the 1997 Kyoto Protocol.

Q.3) Consider the following:

1. Air India

2. BSNL

3. SAIL

Which of the above is/are loss making PSUs of Government of India ?

a) 1 only

b) 1 and 2

c) 2 and 3

d) All of the above

Answer: (d)

The Public Enterprises Survey, tracking the performance of CPSEs in 2015-16, revealed that the top
three loss-making CPSEs -- SAIL, BSNL and Air India -- incurred a loss equal to 51.65 per cent of the
total loss made by the top 10 loss-making CPSEs in 2015-16.

http://www.business-standard.com/article/pti-stories/sail-bsnl-air-india-worst-performing-psus-in-fy16-surv
ey-117032400558_1.html

Q.4) Consider the following:

1. Chitradurga

2. Pashan

3. Wheeler Island

Which of the above is/are test ranges of DRDO ?

PrelimsDailyArchive-Clickhere|TargetMainsArchive-Clickhere

ADailyInitiativefromCivilsdaily|DownloadFREEApp|JointhePrelimsModule

a) 1 and 2

b) 2 only

c) 1 and 3

d) All of the above

Answer: (d)

Challakere Aeronautical Test Range (ATR) is an out-door testing and evaluating facility set up by DRDO
exclusively for unmanned and manned aircraft. Challakere ATR is located in Chitradurga district,
Karnataka. Dr. Abdul Kalam Island, formerly known as Wheeler Island, is an island off the coast of
Odisha, India. The Integrated Test Range missile testing facility is located on the island. Pashan Test
Range is an out-door testing and evaluating facility of Armament Research and Development
Establishment (DRDO) for armament stores. It is located in outskirt of Pune near Pashan village and is
spread over 150 acres of land surrounded by hills on three sides.

Q.5) Consider the following statements:

1. Prithvi is a surface-to-air missile developed by ISRO.

2. It has a strike range of 5000 km.

Which of the above is/are incorrect ?

a) 1 only

b) 2 only

c) Both 1 and 2

d) Neither 1 nor 2

Answer: (c)

Prithvi is a surface-to-surface missile, which has a strike range of 350 km and is developed by DRDO.

PrelimsDailyArchive-Clickhere|TargetMainsArchive-Clickhere

ADailyInitiativefromCivilsdaily|DownloadFREEApp|JointhePrelimsModule

Q.6) Quit India Movement was launched in response to

a) Cabinet Mission Plan

b) Cripps Proposals

c) Simon Commission Report

d) Wavell Plan

Answer: (b)

The Cripps Mission Proposal , which included terms like establishment of Dominion, establishment of a
Constituent Assembly and right of the Provinces to make separate constitutions, These would be,
however, granted after the cessation of the Second World War,So fail to satisfy Indian Nationalists.
According to the Congress this Declaration only offered India a promise that was to be fulfilled in the
future. Commenting on this Gandhi said; 'It is a post dated cheque on a crashing bank and in CWC
meeting at wardha july 1942 accepted the idea of Struggle.

Q.7) The balance of payments of a country is a systematic record of

a) all import and transactions of a during a given period normally a year

b) goods exported from a country during a year

c) economic transaction between the government of one country to another

d) capital movements from one country to another

Answer: (a)

The balance of payments (BoP) record the transactions in goods, services and assets between residents
of a country with the rest of the world for a specified time period typically a year. so best option is A

PrelimsDailyArchive-Clickhere|TargetMainsArchive-Clickhere

ADailyInitiativefromCivilsdaily|DownloadFREEApp|JointhePrelimsModule

Q.8) The Reserve Bank of India regulates the commercial banks in matters of

1. liquidity of assets

2. branch expansion

3. merger of banks

4. windingup of banks

Select the correct answer using the codes given below.

a) 1 and 4 only

b) 2, 3 and 4 only

c) 1, 2 and 3 only

d) 1, 2, 3 and 4

Answer: (d)

All statemets are implied from the fact that RBI is regulator of Banks. One could think that merger might
come under CCI only but its comes both under CCI and RBI. 1. CRR, SLR etc.2. Press Information
Bureau English Releases As per Reserve Bank of Indias (RBI) Branch Authorisation Policy, general
permission has been granted to domestic Scheduled Commercial Banks (other than RRBs) to open
branches / mobile branches / Administrative Offices / CPCs (Service Branches), (i) in Tier 3 to Tier 6
centres (with population up to 49,999) and (ii) in rural, semi-urban and urban centres of the North-Eastern
States and Sikkim subject to reporting. Opening of branches by these banks in Tier 1 and Tier 2 centres
(centres with population of 50,000 and above) requires prior approval of RBI except in North Eastern
States and Sikkim. Banks apply for authorisations for opening of branches in Tier 1 and Tier 2 centres in
their Annual Branch Expansion Plan. 3.

PrelimsDailyArchive-Clickhere|TargetMainsArchive-Clickhere

ADailyInitiativefromCivilsdaily|DownloadFREEApp|JointhePrelimsModule

PRELIMSDAILY
8Questions+1Tikdam/Titbitsinexplanations

----------------------------------------------------------------

7June2017QuestionsandAnswers

Q.1) The Purchasing Managers Index (PMI) is based on some major indicators. Which of the
following is not the major indicator?

a) Inventory levels

b) Production

c) Employment environment

d) All of the above

Answer: (d)

The Purchasing Managers Index (PMI) is an indicator of the economic health of the
manufacturing sector. The PMI is based on five major indicators: new orders, inventory levels,
production, supplier deliveries and the employment environment. The purpose of the PMI is to
provide information about current business conditions to company decision makers, analysts
and purchasing managers.

Q.2) Which of the following countries touch Gulf of Aden?

1. Yemen

2. Qatar

3. Egypt

PrelimsDailyArchive-Clickhere|TargetMainsArchive-Clickhere

ADailyInitiativefromCivilsdaily|DownloadFREEApp|JointhePrelimsModule

4. Somalia

5. Pakistan

Select the correct answer using the codes given below.

a) 1 and 5 only

b) 1, 2 and 3

c) 1 and 4 only

d) 3 only

Answer: (c)

The Gulf of Aden is a gulf located in the Arabian Sea between Yemen, on the south coast of the Arabian
Peninsula, and Somaliland and Somalia in the Horn of Africa.

Q.3) Which of the following disease is not cuased by Virus?

a) Malaria

b) Chikungunya

c) Measles

d) Rabies

Answer: (a)

Malaria is a mosquito-borne infectious disease affecting humans and other animals caused by parasitic
protozoans (a group of single-celled microorganisms) belonging to the Plasmodium type.

PrelimsDailyArchive-Clickhere|TargetMainsArchive-Clickhere

ADailyInitiativefromCivilsdaily|DownloadFREEApp|JointhePrelimsModule

Q.4) "Consider the following statements regarding reign of the Mughal emperor Akbar:

1. He participated in native festivals, realizing that a stable empire depended on the co-operation and
goodwill of his subjects.

2. He strengthened his control on the nobility and the army by introducing the mansabdari system.

Which of the following statements are correct?

a) 1 only

b) 2 only

c) Both 1 and 2

d) Neither 1 nor 2

Answer: (c)

Q.5) Consider the following statements:

1. The Constituent Assembly started functioning in December 1946.

2. Jawaharlal Nehru was the first President of the Constituent Assembly.

3. The Constituent Assembly adopted the Constitution of India on 26th November 1949.

Which of the statements given above are correct?

a) 1 and 2 only

b) 1 and 3 only

c) 3 only

d) 2 and 3 only

Answer: (b)

The Consituent Assembly held its first meeting on 9th December 1946 under the Presidentship of
Sachindand Sinha. In this meeting Rajendra Prasad was elected the President of Constituent Assembly for
the further sittings to be held.

PrelimsDailyArchive-Clickhere|TargetMainsArchive-Clickhere

ADailyInitiativefromCivilsdaily|DownloadFREEApp|JointhePrelimsModule

Q.6) Which of the following diseases can be transmitted from one person to another through tattooing?

1. Chikungunya

2. Hepatitis B

3. HIVAIDS

Select the correct answer using the codes given below.

a) 1 only

b) 2 and 3 only

c) 1 and 3 only

d) 1, 2 and 3

Answer: (b)

Currently there are some 200 communicable diseases that can be transmitted through tattooing.Hepatitis
B:- Hepatitis B and C can lead to serious, permanent liver damage and are known to be transmittable
through tattooing. Hepatitis can survive outside its host for up to two weeks and can resists most normal
forms of disinfection. Also, studies show that the ratio of hepatitis transmission through accidental needle
stick is much greater than that of HIV. About one in five as opposed to one in 200 for HIV.HIV/AIDS:-
The term needle in tattooing is a misnomer since tattoos are not applied by needles but rather with pins. A
needle is typically a hollow core implement that is used primarily for intravenous injections such as a
syringe or IV needle. Tattoo 'needles' are solid core and push the ink into the skin rather than inject it.
This requires that the ink, and any contaminated blood mixed with it, must adhere to the outside surface
of the pins causing the virus to come into contact with the open air.

Q.7) Which of the following statements is/are applicable to Jain doctrine?

1. The surest way of annihilating Karma is to practice penance.

2. Every object, even the smallest particle has a soul.

3. Karma is the bane of the soul and must be ended.

Select the correct answer using the codes given below.

PrelimsDailyArchive-Clickhere|TargetMainsArchive-Clickhere

ADailyInitiativefromCivilsdaily|DownloadFREEApp|JointhePrelimsModule

a) 1 only

b) 2 and 3 only

c) 1 and 3 only

d) 1, 2 and 3

Answer: (d)

The surest way of annihilating Karma is to practice penance. Karma is the bane of the soul and must be
ended. "Swadhyay Paramam Tap" According to Jain doctrines Penance washes away all the blemishes
and purges the soul of all karmic matter. Also, according to Jainism, Karma is the bane of the soul. Karma
not only encompasses the causality of transmigration but it is also conceived as an extremely subtle
matter which infiltrates the soul, obscuring its natural, transparent, pure qualities. Karma is thought of as a
kind of pollution that taints the soul with various colors. Based on its karma, a soul undergoes
transmigration and reincarnation in various states of existence like heavens or hells or as humans or
animals.

Q.8) Which one of the following terms describes not only the physical space occupied by an organism,
but also its functional role in the community of organisms?

a) Ecotone

b) Ecological niche

c) Habitat

d) Home range

Answer: (d)

The ecological niche is the ecological role and space that an organism fills in an ecosystem.The ecological
niche involves both the place where an organism lives and the roles that an organism does in its habitat.
For example, the ecological niche of a sunflower growing in the backyard includes absorbing light, water
and nutrients (for photosynthesis), providing shelter and food for other organisms (e.g. bees, ants, etc.),
and giving off oxygen into the atmosphere.

PrelimsDailyArchive-Clickhere|TargetMainsArchive-Clickhere

ADailyInitiativefromCivilsdaily|DownloadFREEApp|JointhePrelimsModule

PRELIMSDAILY
8Questions+1Tikdam/Titbitsinexplanations

----------------------------------------------------------------

8June2017QuestionsandAnswers

Q.1) Which of the following statements regarding the Indian Ocean Dipole (IOD) is/are correct?

1. IOD is also known as Indian Nino.

2. It buffers the impact of El Nino and contribute to better rains.

Select the correct options using the codes given below.

a) 2 only

b) 1 only

c) Neither 1 nor 2

d) Both are Correct

Answer: (b)

The Indian Ocean Dipole (IOD) is also known as the Indian Nio. IOD is an irregular oscillation of
sea-surface temperatures in which the western Indian Ocean becomes alternately warmer and then
colder than the eastern part of the ocean. The IOD is a swing in surface temperatures that turns the
western Indian Ocean alternately warmer and then colder than the eastern part of the ocean. During
a Positive IOD, the Eastern part of the equatorial Indian Ocean becomes abnormally cool and the
Western part remains unusually warm. The reverse occurs during a Negative IOD. A Positive IOD is
said to be beneficial for the Monsoon, by pushing precipitation towards India. And a Negative IOD
has been found during a couple of drought years. So, it is not necessary that it contribute to better
rains always.

PrelimsDailyArchive-Clickhere|TargetMainsArchive-Clickhere

ADailyInitiativefromCivilsdaily|DownloadFREEApp|JointhePrelimsModule

Q.2) The Unique Identification Authority of India (UIDAI) comes under

a) Ministry of Electronics and Information Technology

b) Ministry of Home Affairs

c) Ministry of Human Resource Development

d) Ministry of Human Resource Development

Answer: (a)

The Unique Identification Authority of India (UIDAI), a statutory authority established in 2016 by the
Government of India, comes under the Ministry of Electronics and Information Technology.

Q.3) Consider the following statements:

1. Capital gains are the profit realized on the sale of a non-inventory asset that was purchased at amount
lower than the amount realized on the sale.

2. Capital gains are non taxable.

Which of the above is/are correct ?

a) 1 only

b) 2 only

c) Both 1 and 2

d) Neither 1 nor 2

Answer: (a)

Capital gains are profits realized on the sale of a non-inventory asset that was purchased at a cost
amount that was lower than the amount realized on the sale. These are taxable and tax is known as
Capital gains tax.

PrelimsDailyArchive-Clickhere|TargetMainsArchive-Clickhere

ADailyInitiativefromCivilsdaily|DownloadFREEApp|JointhePrelimsModule

Q.4) Consider the following:

1. Income tax

2. Wealth tax

3. Corporation tax

4. Capital gains tax

Which of the above is/are direct taxes ?

a) 2 and 3 only

b) 3 and 4 only

c) 1, 2 and 4

d) All of the above

Answer: (a)

All the above mentioned taxes are direct taxes in India.

Q.5) Consider the following statements:

1. The National AIDS Control Organisation (NACO) was established in 1989.

2. It is a division of India's Ministry of Women and Child Development.

3. It provides leadership to HIV/AIDS control programme in India through 35 HIV/AIDS Prevention and
Control Societies.

4. It is an innovation which represented the first time that HIV/AIDS education could be provided in a
curriculum which did not need to be coupled with sex education.

Which of the following is/are correct?

a) 1 only

b) 1,3 and 4 only

c) 3 and 4 only

d) 1,2,3 and 4

PrelimsDailyArchive-Clickhere|TargetMainsArchive-Clickhere

ADailyInitiativefromCivilsdaily|DownloadFREEApp|JointhePrelimsModule

Answer: (b)

The National AIDS Control Organisation (NACO) was established in 1992. It is a division of India's
Ministry of Health and Family Welfare. It provides leadership to HIV/AIDS control programme in India
through 35 HIV/AIDS Prevention and Control Societies.

Q.6) Photochemical smog is a resultant of the reaction among

a) NO2, 03 and peroxyacetyl nitrate in the presence of sunlight

b) CO, 02 and peroxyacetyl nitrate in the presence of sunlight

c) CO, CO2 and N02 at low temperature

d) High concentration of N02, O3 and CO in the evening

Answer: (a)

Major Chemical Pollutants in Photochemical Smog: -1)Nitrogen Oxides(NO and NO2)2)Volatile Organic
Compounds (VOCs) 3)Ozone (O3) 4)Peroxyacetyl Nitrates (PAN)

Q.7) Consider the following minerals

1. Calcium

2. Iron

3. Sodium

Which of the minerals given above is/are required by human body for the contraction of muscles?

a) 1 only

b) 2 and 3 only

c) 1 and 3 only

d) 1, 2 and 3

Answer: (d)

PrelimsDailyArchive-Clickhere|TargetMainsArchive-Clickhere

ADailyInitiativefromCivilsdaily|DownloadFREEApp|JointhePrelimsModule

Calcium:-Calcium is essential for activating enzymes which cause muscle contraction.Iron:- Iron helps
support continued muscle function by supporting energy production in your muscle cells, so that muscle
fibers always have the energy they need to contract properly. Iron also helps muscles store oxygen to fuel
muscle contractions, and promotes healthy circulation so that muscles can get additional oxygen from
bloodstream.Sodium:-One need sodium for muscle contraction because it balances potassium to
maintain membrane potential.

Q.8) Consider the following statements:

The Parliamentary Committee on Public Accounts

1. consists of not more than 25 Members of the Lok Sabha

2. scrutinizes appropriation finance accounts of Government

3. of the Auditor examines the report Comptroller and General of India

Which of the statements given a bove is / are correct?

a) 1 only

b) 2 and 3 only

c) 3 only

d) 1, 2 and 3

Answer: (b)

Statement 1 is wrong as it consists of 22 members (15 from Lok Sabha and 7 from Rajya Sabha)

PrelimsDailyArchive-Clickhere|TargetMainsArchive-Clickhere

ADailyInitiativefromCivilsdaily|DownloadFREEApp|JointhePrelimsModule

PRELIMSDAILY
8Questions+1Tikdam/Titbitsinexplanations

----------------------------------------------------------------

9June2017QuestionsandAnswers
Q.1) Consider the following statements regarding the scientific term 'Genome':

1. Genome contains all of the information needed to build and maintain that organism.

2. The term genome was created in 1920 by Hans Winkler.

3. It is a haploid set of chromosomes.

Which of the following statements given above is/are correct?

a) 1, 2 and 3

b) 1 and 3 only

c) 2 and 3 only

d) 1 and 3 only

Answer: (a)

A genome is an organisms complete set of DNA, including all of its genes. Each genome contains all of
the information needed to build and maintain that organism. In humans, a copy of the entire
genomemore than 3 billion DNA base pairsis contained in all cells that have a nucleus.

PrelimsDailyArchive-Clickhere|TargetMainsArchive-Clickhere

ADailyInitiativefromCivilsdaily|DownloadFREEApp|JointhePrelimsModule

Q.2) Amaq News Agency is related to

a) North Korea

b) Inter-Services Intelligence(ISI)

c) Islamic State of Iraq and the Levant(ISIL)

d) Syria

Answer: (c)

Amaq News Agency is a news outlet linked to the Islamic State of Iraq and the Levant(ISIL). It gets tips
from ISIL and is often the first point of publication for claims of responsibility by the group.

Q.3) Consider the following statements:

1. Repo rate is a tool of liquidity management used only in emergency cases.

2. SLR is the portion of bank deposits that has to be kept with RBI in cash.

Which of the above is/are correct?

a) 1 only

b) 2 only

c) Both 1 and 2

d) Neither 1 nor 2

Answer: (d)

REPO rate is changed from time to time to manage liquidity and not only in emergency conditions. SLR is
portion of bank deposits that have to be invested in government bonds and other such investments like
gold etc. It is CRR that requires cash to be kept with RBI.

PrelimsDailyArchive-Clickhere|TargetMainsArchive-Clickhere

ADailyInitiativefromCivilsdaily|DownloadFREEApp|JointhePrelimsModule

Q.4) Business expectations index is brought out by:

a) Ministry of Commerce

b) Confederation of Indian Industries (CII)

c) Reserve Bank of India

d) Labour Bureau

Answer: (c)

Q.5) The Union Cabinet gave its approval to amend the Prevention of Corrution Act, 1998. Consider the
following statements in this regard:

1. It provides more stringent punishment both for the bribr guver and the bribe taker.

2. Under this, the maximum imprisonment would be 7 years which will bring corruption into the heinous
crime category.

3. The average trial period of cases should complete within two years.

Which of the following are correct?

a) 1 and 2 only

b) 1 and 3 only

c) 2 only

d) 1,2 and 3

Answer: (d)

Prevention of Corrution Act, 1998 provides more stringent punishment both for the bribr guver and the
bribe taker. Under this, the maximum imprisonment would be 7 years whcih will bring corruption into the
heinous crime category. The average trial period of cases should complete within two years.

Q.6) Consider the following Bhakti Saints: (CSE: 2013)

1. Dadu Dayal

PrelimsDailyArchive-Clickhere|TargetMainsArchive-Clickhere

ADailyInitiativefromCivilsdaily|DownloadFREEApp|JointhePrelimsModule

2. Guru Nanak

3. Tyagaraja

Who among the above was/were preaching when the Lodi dynasty fell and Babur took over?

a) 1 and 3

b) 2 only

c) 2 and 3

d) 1 and 2

Answer: (b)

Guru Nanak ( 1469 1539)Dadu Dayal (15441603)Kakarla Tyagabrahmam (1767 1847) Lodi dynasty
fell and Badur took over: 1526

Q.7) With reference to the food chains in ecosystems, which of the following kinds of organism is / are
known as decomposer organism/organisms? (CSE: 2013)

1. Virus

2. Fungi

3. Bacteria

Select the correct answer using the codes given below.

a) 1 only

b) 2 and 3 only

c) 1 and 3 only

d) 1, 2 and 3

Answer: (b)

Most viruses need a living host organism-they are more like a parasite. Viruses reproduce by sticking
their DNA into living cells and their DNA 'takes over' the living cell and uses the living cells' tools to
replicate the virus' DNA and make more viruses. Bacteria that are used in decomposition need the dead
animals' flesh for food.

PrelimsDailyArchive-Clickhere|TargetMainsArchive-Clickhere

ADailyInitiativefromCivilsdaily|DownloadFREEApp|JointhePrelimsModule

Q.8) The most important fishing grounds of the world are found in the regions where (CSE:2013)

a) warm and cold atmospheric currents meet

b) rivers drain out large amounts of fresh water into the sea

c) warm and cold oceanic currents meet

d) continental shelf is undulating

Answer: (c)

Planktons are abundant where warm and cold currents meet. That make these areas important fishing
grounds of the world

PrelimsDailyArchive-Clickhere|TargetMainsArchive-Clickhere

ADailyInitiativefromCivilsdaily|DownloadFREEApp|JointhePrelimsModule

PRELIMSDAILY
8Questions+1Tikdam/Titbitsinexplanations

----------------------------------------------------------------

10June2017QuestionsandAnswers
Q.1) Consider the following statements regarding the The Regional Anti-Terrorist Structure (RATS):

1. It is headquartered in Tashkent, Uzbekistan.

2. The Head of RATS is elected to a three-year term.

3. It is a is a temporary organ of the SCO.

Which of the following statements given above is/are correct?

a) 1, 2 and 3

b) 1 and 3 only

c) 1 and 2 only

d) 2 and 3 only

Answer: (c)

The Regional Anti-Terrorist Structure (RATS), headquartered in Tashkent, Uzbekistan, is a permanent


organ of the SCO which serves to promote cooperation of member states against the three evils of
terrorism, separatism and extremism. The Head of RATS is elected to a three-year term. Each member
state also sends a permanent representative to RATS.

Q.2) Consider the following statements:

1. Lowering the SLR may result in lowering of lending rate by banks.

2. SLR is meant to be used for private sector investment.

Which of the above is/are correct?

PrelimsDailyArchive-Clickhere|TargetMainsArchive-Clickhere

ADailyInitiativefromCivilsdaily|DownloadFREEApp|JointhePrelimsModule

a) 1 only

b) 2 only

c) Both 1 and 2

d) Neither 1 nor 2

Answer: (a)

Explanation- Lowering the SLR would provide banks with more financial resources to lend and this may
lead to lowering of interest rates.

SLR is used by government and most of it is invested in government's securities. It is not meant for
private sector use.

Q.3) Real GDP is:

a) evaluated at market prices of current year

b) evaluated at market prices of some base year

c) total current revenue from all types of taxes net of transfer payments

d) Difference between exports and imports

Answer: (b)

Explanation- Real GDP is evaluated at market prices of some base year (not at current prices) in order to
negate the effects of inflation/deflation in GDP calculations.

Q.4) Consider the following statements:

1. Keratitis is the inflammation of the eye

2. Keratitis can be caused by both bacteria and fungi

3. Fungi attach themselves to the cornea and release enzymes that break down the corneal proteins for
their nutritional requirements

Which of the statements given above is/are incorrect?

PrelimsDailyArchive-Clickhere|TargetMainsArchive-Clickhere

ADailyInitiativefromCivilsdaily|DownloadFREEApp|JointhePrelimsModule

a) 1, 2 and 3

b) 2 and 3

c) 1 and 3

d) None of the above

Answer: (d)

Keratitis is the inflammation of the eye. Keratitis can be caused by both bacteria and fungi. Fungi attach
themselves to the cornea and release enzymes that break down the corneal proteins for their nutritional
requirements.

Q.5) Consider the following statements:

1. The CPCB of India is a statutory organisation under the Ministry of Environment, Forest and Climate
Change (MoEF&CC)

2. CPCB is entrusted with the powers and functions under the Air (Prevention and Control of Pollution)
Act, 1981 but not under Water (Prevention and Control of Pollution) Act, 1974

3. It is responsible for maintaining national standards under a variety of environmental laws, in


consultation with zonal offices, tribal, and local governments

Which of the statements given above is/are correct?

a) 1 only

b) 2 and 3

c) 1 and 3

d) 3 only

Answer: (c)

The CPCB of India is a statutory organisation under the Ministry of Environment, Forest and Climate
Change (MoEF&CC). CPCB is entrusted with the powers and functions under the Air (Prevention and
Control of Pollution) Act, 1981 and also under Water (Prevention and Control of Pollution) Act, 1974. It is
responsible for maintaining national standards under a variety of environmental laws, in consultation with
zonal offices, tribal, and local governments.

PrelimsDailyArchive-Clickhere|TargetMainsArchive-Clickhere

ADailyInitiativefromCivilsdaily|DownloadFREEApp|JointhePrelimsModule

Q.6) Which of the following is/are unique characteristic/characteristics of equatorial forests? (CSE: 2013)

1. Presence of tall, closely set trees with crowns forming a continuous canopy

2. Coexistence of a large number of species

3. Presence of numerous varieties of epiphytes

Select the correct answer using the code given below:

a) 1 only

b) 2 and 3 only

c) 1 and 3 only

d) 1, 2 and 3

Answer: (d)

The canopy is the primary layer of the forest forming a roof over the two remaining layers. The densest of
the biodiversity is found here along with a large variety of epiphytes.

Q.7) Which of the following constitute Capital Account? (CSE:2013)

1. Foreign Loans

2. Foreign Direct Investment

3. Private Remittances

4. Portfolio Investment

Select the correct answer using the codes given below.

a) 1, 2 and 3

b) 1, 2 and 4

c) 2, 3 and 4

d) 1, 3 and 4

Answer: (b)

PrelimsDailyArchive-Clickhere|TargetMainsArchive-Clickhere

ADailyInitiativefromCivilsdaily|DownloadFREEApp|JointhePrelimsModule

Private Remittances forms a part of current account and not capital account.

Q.8) Consider the following historical places: (CSE: 2013)

1. Ajanta Caves

2. Lepakshi Temple

3. Sanchi Stupa

Which of the above places is / are also known for mural paintings?

a) 1 only

b) 1 and 2 only

c) 1, 2 and 3

d) None

Answer: (b)

The Lepakshi temple has the finest specimens of mural paintings of the Vijayanagar kings.

PrelimsDailyArchive-Clickhere|TargetMainsArchive-Clickhere

ADailyInitiativefromCivilsdaily|DownloadFREEApp|JointhePrelimsModule

PRELIMSDAILY
8Questions+1Tikdam/Titbitsinexplanations

----------------------------------------------------------------

12June2017QuestionsandAnswers
Q.1) Consider the following statements regarding the structure and working of the Nuclear Fission
Reactor:

1. Control rods in the nuclear fission reactor are made of a neutron poison.

2. Nuclear Fission products decay via alpha decay.

Which of the following statements given above is/are correct?

a) 1 only

b) 2 only

c) Both are correct

d) None is correct

Answer: (a)

A nuclear reactor coolant usually water but sometimes a gas or a liquid metal (like liquid sodium) or
molten salt is circulated past the reactor core to absorb the heat that it generates. The heat is carried
away from the reactor and is then used to generate steam. Most reactor systems employ a cooling
system that is physically separated from the water that will be boiled to produce pressurized steam for the
turbines, like the pressurized water reactor. However, in some reactors the water for the steam turbines is
boiled directly by the reactor core; for example the boiling water reactor.

The initial fission products may be unstable and typically undergo beta decay to move towards a stable
configuration, converting a neutron to a proton with each beta emission. (Fission products do not decay
via alpha decay.)

PrelimsDailyArchive-Clickhere|TargetMainsArchive-Clickhere

ADailyInitiativefromCivilsdaily|DownloadFREEApp|JointhePrelimsModule

Q.2) Consider the following statements:

1. Land acquisition act 2013 requires farmer's/land owner's consent in order to acquire land for various
purposes.

2. No amendments have been proposed to this act till date.

Which of the above is/are correct?

a) 1 only

b) 2 only

c) Both 1 and 2

d) Neither 1 nor 2

Answer: (a)

The LARR Act, 2013 outlines the process to be followed when land is acquired for a public purpose. The
LARR Act, 2013 requires that the consent of 80% of land owners is obtained for private projects and that
the consent of 70% of land owners be obtained for PPP projects. The Bill exempts the five categories(i)
defence, (ii) rural infrastructure, (iii) affordable housing, (iv) industrial corridors, and (v) infrastructure
projects including Public Private Partnership (PPP) projects where the government owns the land from
this provision.

Amendment(s) have been proposed and ordinance was also promulgated in 2014 but it had to be taken
back due to strong opposition by various stakeholders.

Q.3) Consider the following statements:

1. Poor families eligible under the Rashtriya Swasthya Beema Yojana can also avail facilities in ESI
hospitals and dispensaries.

2. ESI is a self-financing social security and health insurance scheme for Indian as well as foreign
workers

Which of the above is are/correct?

a) 1 only

b) 2 only

PrelimsDailyArchive-Clickhere|TargetMainsArchive-Clickhere

ADailyInitiativefromCivilsdaily|DownloadFREEApp|JointhePrelimsModule

c) Both 1 and 2

d) Neither 1 nor 2

Answer: (a)

Explanation- In addition to insured workers, poor families eligible under the RSBY can also avail facilities
in ESI hospitals and dispensaries. ESI is a self-financing social security and health insurance scheme for
Indian workers only (and not foreign).

Q.4) COnsider the following statements:

1. The Comptroller and Auditor General of India can be removed by an address from both the Houses of
the Parliament only.

2. The CAG of India has no control over the issue of money from the consolidated fun of India.

Which of the following options is correct?

a) 1 only

b) 2 only

c) 1 and 2 both

d) Neither 1 nor 2

Answer: (c)

The Comptroller and Auditor General of India can be removed by an address from both the Houses of the
Parliament only. The CAG of India has no control over the issue of money from the consolidated fund of
India.

Q.5) Consider the following statements about Shuddhi movement:

1. It was started by Swami Dayanand Saraswati and Swami Shraddhanand.

2. It was aimed to accept people back to Hinduism.

3. It was aimed at abolishing the practise of untouchability.

Which of the statements given above is/are correct?

PrelimsDailyArchive-Clickhere|TargetMainsArchive-Clickhere

ADailyInitiativefromCivilsdaily|DownloadFREEApp|JointhePrelimsModule

a) 1 and 2 only

b) 1 and 3 only

c) 2 only

d) 1,2 and 3

Answer: (d)

Q.6) With reference to the history of philosophical thought in India, consider the following statements
regarding Sankhya school: (CSE:2013)

1 . Sankhya does not accept the theory of rebirth or transrmigration of soul.

2. Sankhya holds that it is the selfknowledge that leads to liberation and not any exterior influence or
agent.

Which of the statements given above is /are correct?

a) 1only

b) 2 only

c) Both 1 and 2

d) Neither 1 nor 2

Answer: (b)

Samkhya is an Orthodox Indian philosophical system. Orthodox Indian philosophical system believes in
soul and karma theory. For Karma theory to hold, soul has to undergo rebirth or transmigration.
Therefore, without liberation no soul can be free from the cycle of rebirth or transmigration.Samkhya
school philosophy- after you have died in this life you will go through a process of rebirth where
depending on your actions in your former life your status will be decided.

Q.7) In the context of India, which of the following principles is/are implied institutionally in the
parliamentary government? (CSE: 2013)

1. Members of the Cabinet are Members of the Parliament.

2. Ministers hold the office till they enjoy confidence in the Parliament.

PrelimsDailyArchive-Clickhere|TargetMainsArchive-Clickhere

ADailyInitiativefromCivilsdaily|DownloadFREEApp|JointhePrelimsModule

3. Cabinet is headed by the Head of the State.

Select the correct answer using the codes given below.

a) 1 and 2 only

b) 3 only

c) 2 and 3 only

d) 1, 2 and 3

Answer: (a)

statement 3 is wrong as Cabinet is headed by Head of Government (i.e.PrimeMinister) and not by Head
of State (i.e. President).

Q.8) The annual range of temperature in the interior of the continents is high as compared to coastal
areas. What is / are the reason / reasons?

1. Thermal difference between land and water

2. Variation in altitude between continents and oceans

3. Presence of strong winds in the interior

4. Heavy rains in the interior as compared to coasts

Select the correct answer using the codes given below.

a) 1 only

b) 1 and 2 only

c) 2 and 3 only

d) 1, 2, 3 and 4

Answer: (a)

Land heats up and cool down very fast as compared to the water. Areas in the interior of India are far
away from the moderating influence of the sea. Such areas have extremes of climate. Statement 2 is
wrong as there may or may not be much difference between the altitudes of land and ocean. Statment 3

PrelimsDailyArchive-Clickhere|TargetMainsArchive-Clickhere

ADailyInitiativefromCivilsdaily|DownloadFREEApp|JointhePrelimsModule

is wrong as winds do not contribute enough to the variations in temperature. Statement 4 is wrong as
coastal region recieve more rainfall.

PrelimsDailyArchive-Clickhere|TargetMainsArchive-Clickhere

ADailyInitiativefromCivilsdaily|DownloadFREEApp|JointhePrelimsModule

PRELIMSDAILY
8Questions+1Tikdam/Titbitsinexplanations

----------------------------------------------------------------

13June2017QuestionsandAnswers
Q.1) Which of the following Institutions given below has/have the contempt powers in India?

1. Election Commission of India(EC)

2. National Human Rights Commission (NHRC)

3. Supreme Court of India.

Select the correct options using the codes given below.

a) 3 only

b) 1 and 3 only

c) 1 and 2 only

d) 1, 2 and 3

Answer: (a)

Recently, the EC has written to the Law Ministry, seeking powers to act against those questioning the poll
panels credibility through unfounded allegations. The Commission has sought amendments to the
Contempt of Courts Act, 1971. This is to empower it to punish anyone being disobedient or discourteous
towards its authority.

Q.2) India and Israel are negotiating for a deal of Spike Missile. What kind of missile is it?

a) Surface to Air Missile

b) Cruise Missile

c) Anti Tank Missile

PrelimsDailyArchive-Clickhere|TargetMainsArchive-Clickhere

ADailyInitiativefromCivilsdaily|DownloadFREEApp|JointhePrelimsModule

d) Torpedo

Answer: (c)

Spike is an Israeli fourth generation man-portable fire-and-forget anti-tank guided missile and
anti-personnel missile with a tandem-charge HEAT warhead, developed and designed by the Israeli
company Rafael Advanced Defense Systems.

Q.3) Which of the following statements is/are correct regarding the Budgetary terms often seen in the
News?

1. A fiscal deficit occurs when a government's total expenditures exceed the revenue that it generates,
including money from borrowing.

2. A revenue deficit occurs when the net income generated, revenues less expenditures, falls short of the
projected net income.

Select the correct option using the codes given below.

a) 1 only

b) 2 only

c) Both are correct

d) None is correct

Answer: (b)

A fiscal deficit occurs when a government's total expenditures exceed the revenue that it generates,
excluding money from borrowings. Deficit differs from debt, which is an accumulation of yearly deficits.
Revenue deficit happens when the actual amount of revenue received and/or the actual amount of
expenditures do not correspond with budgeted revenue and expenditure figures. This is the opposite of a
revenue surplus, which occurs when the actual amount of net income exceeds the projected amount.

PrelimsDailyArchive-Clickhere|TargetMainsArchive-Clickhere

ADailyInitiativefromCivilsdaily|DownloadFREEApp|JointhePrelimsModule

Q.4) What is the religion of Madhesi people living in Nepal whose name are often seen in the news?

a) Hindu

b) Muslim

c) Christianity

d) No Definite Religion

Answer: (d)

The term Madhesi people is ambiguous. Anthropologists use the term for people of Indian ancestry
residing in the Terai of Nepal and comprising various cultural groups such as Hindu caste groups,
muslims, merchants and indigenous people of the Terai. Many of these groups share cultural traditions
and marital ties with people living south of the international border in Bihar, Uttar Pradesh and West
Bengal. In recent times, some politicians and journalists use the term for all Nepali citizens of the Terai.

Q.5) Which of the following taxes will be bound together by the GST?

1. Central Excise Duty

2. Service Tax

3. Food Tax

4. Entertainment Tax

5. Taxes applicable on lotteries

Select the correct options using the codes given below.

a) All of the Above

b) 1, 2 and 3 only

c) 3, 4 and 5 only

d) 1, 4 and 5 only

Answer: (a)

PrelimsDailyArchive-Clickhere|TargetMainsArchive-Clickhere

ADailyInitiativefromCivilsdaily|DownloadFREEApp|JointhePrelimsModule

The following taxes will be bound together by the GST: Central Excise Duty, Service Tax, Commercial
Tax, Value Added Tax (VAT), Food Tax, Central Sales Tax (CST), Octroi, Entertainment Tax, Entry Tax,
Purchase Tax, Luxury Tax, Advertisement taxes, Taxes applicable on lotteries.

Q.6) Which of the following is / are the characteristic/ characteristics of Indian coal?

1. High ash content

2. Low sulphur content

3. Low ash fusion temperature

Select the correct answer using the codes given below.

a) 1 and 2 only

b) 2 only

c) 1 and 3 only

d) 1, 2 and 3

Answer: (a)

The most significant characteristic of Indian coal is its high ash content, which varies from 35 to 45 per
cent, compared with that of coal in other parts of the world, which is around 15 per cent. Besides high ash
content, another reason for entrained gasifiers (that operate at higher temperatures than fluidised bed
gasifiers), commonly used in IGCC plants abroad, being not suitable for Indian coal is its high ash fusion
temperature of about 1,500 However, Indian coals sulphur content is low, about 0.5 per cent.

Q.7) Which of the following statements regarding laterite soils of India are correct?

1. They are generally red in colour.

2. They are rich In nitrogen and potash.

3. Tiley are welldeveloped in Rajasthan and UP.

4. Tapioca and cashew nuts grow well on these soils.

Select the correct answer using the codes given below.

PrelimsDailyArchive-Clickhere|TargetMainsArchive-Clickhere

ADailyInitiativefromCivilsdaily|DownloadFREEApp|JointhePrelimsModule

a) 1, 2 and 3

b) 2, 3 and 4

c) 1 and 4

d) 2 and 3 only

Answer: (c)

Statement 2 is wrong as these soils are poor in nitrogen and phosphate. Statement 3 is wrong as laterite
is not found in UP or in Rajasthan. Red laterite soils in Tamil Nadu, AndhraPradesh and Kerala are more
suitable for tree crops like cashewnut.

Q.8) Consider the following statements:

1. Natural gas occurs in the Gondwana beds.

2. Mica occurs in abundance in Kodarma.

3. Dharwars are famous for petroleum.

Which of the statements given above is/are correct?

a) 1 and 2

b) 2 only

c) 2 and 3

d) None

Answer: (b)

Natural gas is obtained alongwith oil in all the oil fields but exclusive reserves have been located along
the eastern coast as well as (Tamil Nadu, Orissa and Andhra Pradesh), Tripura, Rajasthan and off-shore
wells in Gujarat and Maharashtra.Crude petroleum occurs in sedimentary rocks of the tertiary period.

PrelimsDailyArchive-Clickhere|TargetMainsArchive-Clickhere

ADailyInitiativefromCivilsdaily|DownloadFREEApp|JointhePrelimsModule

PRELIMSDAILY
8Questions+1Tikdam/Titbitsinexplanations

----------------------------------------------------------------

14June2017QuestionsandAnswers
Q.1) Under the provisions of the Maintenance and Welfare of Parents and Senior Citizens Act, 2007,
which of the following act as Tribunal?

a) Judge of the High Court

b) Member of National Human Rights Commission(NHRC)

c) Subdivisional Magistrate (SDM)

d) District collector

Answer: (c)

Under the Maintenance and Welfare of Parents and Senior Citizens Act, 2007, parents or the elderly can
make an application to the subdivisional magistrate (SDM) against their children or relatives who are to
inherit their property. In this case, the SDM acts as a tribunal, passes an order directing the children or
relatives to pay the monthly allowance.

Q.2) Which of the following crops are not covered under the Minimum Support Price in Indian Farming
Sector?

1. Maize

2. Jowar

3. Ragi

4. Barley

5. Raw jute

PrelimsDailyArchive-Clickhere|TargetMainsArchive-Clickhere

ADailyInitiativefromCivilsdaily|DownloadFREEApp|JointhePrelimsModule

Select the correct option using the codes given below.

a) 1, 2 and 3 only

b) 2, 3, 4 and 5 only

c) 1, 3 and 4 only

d) All of the above

Answer: (d)

26 commodities are currently covered. They are as follows:

Cereals (7) - paddy, wheat, barley, jowar, bajra, maize and ragi, Pulses (5) - gram, arhar/tur, moong, urad
and lentil, Oilseeds (8) - groundnut, rapeseed/mustard, toria, soyabean, sunflower seed, sesamum,
safflower seed and nigerseed, Copra, De-husked coconut, Raw cotton, Raw jute, Sugarcane (Fair and
remunerative price), Virginia flu cured (VFC) tobacco.

Q.3) Which of the following is not a fertilizer used in Agriculture?

a) Diammonium phosphate

b) Ammonium Sulphate

c) Potash

d) Benzopyrene

Answer: (d)

Benzopyrene is a soil pollutant(and not fertilizer). Soil contamination or soil pollution as part of land
degradation is caused by the presence of chemicals or other alteration in the natural soil environment. It
is typically caused by industrial activity, agricultural chemicals, or improper disposal of waste. The most
common chemicals involved are petroleum hydrocarbons, polynuclear aromatic hydrocarbons (such as
naphthalene and benzo(a)pyrene), solvents, pesticides, lead, and other heavy metals.

PrelimsDailyArchive-Clickhere|TargetMainsArchive-Clickhere

ADailyInitiativefromCivilsdaily|DownloadFREEApp|JointhePrelimsModule

Q.4) Consider the following statements regarding the Consumer Price Index:

1. Consumer Price Index is used in calculation of Dearness Allowance(of Government Employees).

2. The Base year of the CPI is 2012.

Which of the following statements given above is/are correct?

a) 1 only

b) None is correct

c) Both are correct

d) 2 only

Answer: (a)

Consumer Price Index (CPI) in India comprises multiple series classified based on different economic
groups. There are four series, viz the CPI UNME (Urban Non-Manual Employee), CPI AL (Agricultural
Labourer), CPI RL (Rural Labourer)and CPI IW (Industrial Worker). While the CPI UNME series is
published by the Central Statistical Organisation, the others are published by the Department of
Labour.From February 2011 the CPI (UNME) released by CSO is replaced as CPI (urban),CPI (rural) and
CPI (combined). Consumer Price Index is used in calculation of Dearness Allowance which forms an
integral part of salary of a Government Employee.

Q.5) What will be the possible effect of increased Cash Reserve Ratio(CRR) on the Consumer Price
Index(CPI), if all other factors remain unchanged?

a) It will increase

b) It will decrease

c) No Effect

d) Increases upto a threshold limit and then it will freeze.

Answer: (b)

Cash Reserve Ratio (CRR) is a specified minimum fraction of the total deposits of customers, which
commercial banks have to hold as reserves either in cash or as deposits with the central bank. CRR is set
according to the guidelines of the central bank of a country. CPI is inversely proportional to the CRR.

PrelimsDailyArchive-Clickhere|TargetMainsArchive-Clickhere

ADailyInitiativefromCivilsdaily|DownloadFREEApp|JointhePrelimsModule

Q.6) Consider the following crops

1.Cotton

2. Groundnut

3. Rice

4.Wheat

Which of these are Kharif crops?

a) 1 and 4

b) 2 and 3 only

c) 1, 2 and 3

d) 2, 3 and 4

Answer: (c)

Rice is the main kharif crop and groundnut and cotton are also the kharif crops wheat is a raki.

Q.7) "Climate is extreme, rainfall is scanty and the people used to be nomadic herders."

The above statement best describes which of the following regions?

a) African Savannah

b) Central Asian Steppe

c) North American Prairie

d) Siberian Tundra

Answer: (b)

Steppe's climate is continental with extremes of temp. Nomadic tribes like Kirghiz are found. Rainfall is
expected to be light. African Savannah gets moderate rainfall.

PrelimsDailyArchive-Clickhere|TargetMainsArchive-Clickhere

ADailyInitiativefromCivilsdaily|DownloadFREEApp|JointhePrelimsModule

Q.8) Consider the following statements :

1. Inflation benefits the debtors.

2. Inflation benefits the bondholders.

Which of the statements given above is/are correct?

a) 1 only

b) 2 only

c) Both 1 and 2

d) Neither 1 nor 2

Answer: (a)

For debtors, 'real' interest rate goes down with inflation. Thus it benefits them on the other hand, with
inflation the yield of bonds goes down, thus a negative effect for bond-holders.

PrelimsDailyArchive-Clickhere|TargetMainsArchive-Clickhere

ADailyInitiativefromCivilsdaily|DownloadFREEApp|JointhePrelimsModule

PRELIMSDAILY
8Questions+1Tikdam/Titbitsinexplanations

----------------------------------------------------------------

15June2017QuestionsandAnswers
Q.1) Which of the following images are not printed on the new Rs. 500 currency notes?

a) Swachh Bharat logo

b) image of the Red Fort

c) Ashoka Pillar emblem

d) None of the above

Answer: (d)

In the existing series of Rs. 500 banknotes, the inset letter E can be seen in the two number
panels at the upper-left and lower-right corners of the front side. The Rs. 500 notes feature
intaglio printing of Mahatma Gandhi portrait, Ashoka Pillar emblem, bleed lines, circle with Rs.
500 in the right and an identification mark. The notes bear the signature of RBI Governor Urjit
Patel on the front side. The year of printing and the Swachh Bharat logo is printed on the
reverse side. The Rs. 500 banknotes also contain an image of Red Fort on the reverse side.

PrelimsDailyArchive-Clickhere|TargetMainsArchive-Clickhere

ADailyInitiativefromCivilsdaily|DownloadFREEApp|JointhePrelimsModule

Q.2) Recently, US based Laser Interferometer Gravitational Wave Observatory (LIGO) detectors
have picked up signals of yet another merger of two black holes. Consider the following
statements regarding the LIGO:

1. It can also see electromagnetic radiations.

2. It doesn't need to be dish-shaped like telescope mirrors or radio dishes.

Which of the statements given above is/are correct?

a) 1 only

b) Both are correct

c) Neither 1 nor 2

d) 2 only

Answer: (d)

Unlike optical or radio telescopes, LIGO cannot see electromagnetic radiation (e.g., visible light,
radio waves, microwaves) nor does it have to because gravitational waves are not part of the
electromagnetic spectrum. In fact, electromagnetic radiation from space is so unimportant to
LIGO that it is completely isolated and sheltered from the outside world. LIGO cannot (nor does
it need to) see anything. Rather, it 'feels' for invisible gravitational waves.

Q.3) Consider the following statements regardingt the Food Corporation of india(FCI):

1. There is no limit for procurement in terms of volume by the FCI.

2. One of the objectives of the FCI is to regulate market price to provide foodgrains to
consumers at a reliable price.

Which of the statements given above is/are correct?

a) None is correct

b) Both are correct

c) 2 only

PrelimsDailyArchive-Clickhere|TargetMainsArchive-Clickhere

ADailyInitiativefromCivilsdaily|DownloadFREEApp|JointhePrelimsModule

d) 1 only

Answer: (b)

Under the Food Corporations Act 1964 to implement the following objectives of the National
Food Policy :

1. Effective price support operations for safeguarding the interests of the farmers

2. Distribution of foodgrains throughout the country for Public Distribution System

3. Maintaining satisfactory level of operational and buffer stocks of foodgrains to ensure


National Food Security

4. Regulate market price to provide foodgrains to consumers at a reliable price

Q.4) Recently, the Securities and Exchange Board of India (SEBI) has allowed commodity
exchanges to introduce options trading. What is this 'Option'?

a) Share

b) Mutual Fund

c) Contract

d) Debenture

Answer: (c)

In finance, an option is a contract which gives the buyer (the owner or holder of the option) the
right, but not the obligation, to buy or sell an underlying asset or instrument at a specific strike
price on a specified date, depending on the form of the option.

PrelimsDailyArchive-Clickhere|TargetMainsArchive-Clickhere

ADailyInitiativefromCivilsdaily|DownloadFREEApp|JointhePrelimsModule

Q.5) Which of the following elements are used in making Atomic Clocks?

1.Rubidium

2. Cesium

3. Neptunium

Select the correct option using the codes given below.

a) 1 and 2 only

b) 1 and 3 only

c) 2 only

d) 2 and 3 only

Answer: (a)

Rubidium and Cesium are used in making Atomic Clocks.

Q.6) Disguised unemployment generally means

a) large number of people remain unemployed

b) alternative employment is not available

c) marginal productivity of labour is zero

d) productivity of workers is low

Answer: (c)

Disguised unemployment, means that more people are engaged in a job which can be done
with less people, normally seen in agriculture where whole family is involved. It means extra
people can be removed without affecting the productivity, i.e. their marginal productivity is zero.

PrelimsDailyArchive-Clickhere|TargetMainsArchive-Clickhere

ADailyInitiativefromCivilsdaily|DownloadFREEApp|JointhePrelimsModule

Q.7) Consider the following statements:

1. The Council of Ministers in the Centre shall be collectively responsible to the Parliament.

2. The Union Ministers shall hold the office during the pleasure of the President of India.

3. The Prime Minister shall communicate to the President about the proposals for Iegislation.

Which of the Statements given above is/are correct?

a) 1 only

b) 2 and 3 only

c) 1 and 3 only

d) 1,2 and 3

Answer: (b)

statement 2 is from article 75statement 3 is from article 78statement 1 is from article 75 - where
it is collectively responsible to the 'House of People' (Lok Sabha). Parliament means both Rajya
Sabha and Lok Sabha.So answer should be (b)

PrelimsDailyArchive-Clickhere|TargetMainsArchive-Clickhere

ADailyInitiativefromCivilsdaily|DownloadFREEApp|JointhePrelimsModule

Q.8) Consider the following statements:

1. National Development Council is an organ of the Planning Commission.

2. The Economic and Social Planning is kept in the Concurrent List in the Constitution of India.

3. The Constitution of India prescribes that Panchayats should be assigned the task of
preparation of plans for economic development and social justice.

Which of the statements given above is/are correct?

a) 1 only

b) 2 and 3 only

c) 1 and 3 only

d) 1,2 and 3

Answer: (b)

NDC is listed as an advisory body to Planning commission. Moreover the plans made by
Planning commission are placed before NDC for itsacceptance. So statement 1 is wrong.

PrelimsDailyArchive-Clickhere|TargetMainsArchive-Clickhere

ADailyInitiativefromCivilsdaily|DownloadFREEApp|JointhePrelimsModule

PRELIMSDAILY
8Questions+1Tikdam/Titbitsinexplanations

----------------------------------------------------------------

16June2017QuestionsandAnswers
Q.1) Which of the following is/are not the products of ChinaPakistan Economic Corridor?

1. Airports

2. Energy Production

3. SEZ(Special Economic Zones)

Select the correct option using the codes given below.

a) 1 and 3 only

b) 2 and 3 only

c) 1 only

d) 2 only

Answer: (c)

Products: Roadways, railways, Special Economic Zones, energy production, mass transit.

Q.2) Which of the following is/are not the All India Fanancial Institutions regulated by the Reserve Bank of
India?

1. Import Bank of India (Exim Bank)

2. National Bank for Agriculture and Rural Development (NABARD)

3. Small Industries Development Bank of India (SIDBI)

4. National Housing Bank (NHB)

Select the correct option using the codes given below.

PrelimsDailyArchive-Clickhere|TargetMainsArchive-Clickhere

ADailyInitiativefromCivilsdaily|DownloadFREEApp|JointhePrelimsModule

a) All of the above

b) 1 and 4 only

c) 2 and 3 only

d)1, 2 and 4 only

Answer: (b)

According to Economic Survey 2012-13, at the end of March 2012, there were four institutions regulated
by Reserve Bank of India as all-India Financial Institutions: Export - Import Bank of India (Exim Bank),
National Bank for Agriculture and Rural Development (NABARD), Small Industries Development Bank of
India (SIDBI), National Housing Bank (NHB).

Q.3) Consider the following statements regarding the "Insolvency and Bankruptcy Code":

1. The Code establishes the Insolvency and Bankruptcy Board of India.

2. The Code proposes two separate tribunals to oversee the process of insolvency resolution, for
individuals and companies.

Which of the following statements given above is/are correct?

a) 1 only

b) Both are correct

c) 2 only

d) None is correct

Answer: (b)

The Code proposes two separate tribunals to oversee the process of insolvency resolution, for individuals
and companies: (i) the National Company Law Tribunal for Companies and Limited Liability Partnership
firms; and (ii) the Debt Recovery Tribunal for individuals and partnerships.

Q.4) Which of the following terrorist organisation is a Nigeria based terror group?

a) Hamas

b) FARC

c) Boko Haram

PrelimsDailyArchive-Clickhere|TargetMainsArchive-Clickhere

ADailyInitiativefromCivilsdaily|DownloadFREEApp|JointhePrelimsModule

d) THE Lord's Resistance Army

Answer: (c)

Boko Haram is an Islamist based militant group in Nigeria, and is affiliated to and receives funding from
Al-Qaeda. The group was originally established as a Sunni Islamic group and advocates for the
proliferation of the very intolerant Sharia law in places where they have seized power.

Q.5) Consider the following statements regarding the "Goods and Services Tax Network (GSTN)":

1. It is a nonprofit organization.

2. There is no share of Indian States in the GSTN.

Which of the statements given above is/are correct?

a) 2 only

b) None is correct

c) Both are correct

d) 1 only

Answer: (d)

Goods and Services Tax Network (GSTN) is a nonprofit organization formed to create a platform for all
the concerned parties i.e. stakeholders, government, taxpayers to collaborate on a single portal. The
portal will be accessible to the central government which will track down every transaction on its end while
the taxpayers will be having a vast service to return file their taxes and maintain the details. The IT
network will be developed by private firms which are being in tie up with the central government and will
be having stakes accordingly. The known authorized capital of GSTN is Rs. 10 crore (US$1.6 million) in
which Central Government holds 24.5 percent of shares while the state government holds 24.5 percent
and rest with private banking firms.

Q.6) Consider the following statements:

1. The Chairman and the Deputy Chairman of the Rajya Sabha are not the members of that House.

2. While the nominated members of the two Houses of the Parliament have no voting right in the
presidential election, they

have the right to vote in the election of the Vice President.

Which of the statements given above is/are correct?

PrelimsDailyArchive-Clickhere|TargetMainsArchive-Clickhere

ADailyInitiativefromCivilsdaily|DownloadFREEApp|JointhePrelimsModule

a) 1 only

b) 2 only

c) Both 1 and 2

d) Neither 1 nor 2

Answer: (b)

Deputy Chairman of Rajya Sabha is elected by Rajya Sabha from amongits members. So Deputy
Chairman is member of that house.

Q.7) With reference to National Legal Services Authority, consider the following statements :

1.Its objective is to provide free and competent legal services to the weaker sections of the society on the
basis of equal opportunity.

2.It issues guidelines for the State Legal Services Authorities to implement the legal programmes and
schemes throughout the

country.

Which of the statements given above is / are correct?

a) 1 only

b) 2 only

c) Both 1 and 2

d) Neither 1 nor 2

Answer: (c)

The National Legal Services Authority (NALSA) has been constituted under the Legal Services Authorities
Act, 1987 to provide free Legal Services to the weaker sections of the society and to organize Lok Adalats
for amicable settlement of disputes. In every state, State Legal Services Authority has been constituted to
give effect to the policies and directions of the NALSA and to give free legal services to the people and
conduct Lok Adalats in the State.

Q.8) During a thunderstorm, the thunder in the skies is produced by the

1. meeting of cumulonimbus clouds in the sky

2. lightning that separates the nimbus clouds

PrelimsDailyArchive-Clickhere|TargetMainsArchive-Clickhere

ADailyInitiativefromCivilsdaily|DownloadFREEApp|JointhePrelimsModule

3. violent upward movement of air and water particles

Select the correct answer using the codes given below.

a) 1 only

b) 2 and 3

c) 1 and 3

d) None of the above produces the thunder

Answer: (d)

Thunder is the sound caused by lightning. The sudden increase in pressure and temperature from
lightning produces rapid expansion of the air surrounding and within a bolt of lightning. In turn, this
expansion of air creates a sonic shock wave, similar to a sonic boom, which produces the sound of
thunder The cause of thunder has been the subject of centuries of speculation and scientific inquiry. The
first recorded theory is attributed to the Greek philosopher Aristotle in the third century BC, and an early
speculation was that it was caused by the collision of clouds. Subsequently, numerous other theories
were proposed. By the mid-19th century, the accepted theory was that lightning produced a vacuum. In
the 20th century a consensus evolved that thunder must begin with a shock wave in the air due to the
sudden thermal expansion of the plasma in the lightning channel.

PrelimsDailyArchive-Clickhere|TargetMainsArchive-Clickhere

ADailyInitiativefromCivilsdaily|DownloadFREEApp|JointhePrelimsModule

PRELIMSDAILY
8Questions+1Tikdam/Titbitsinexplanations

----------------------------------------------------------------

17June2017QuestionsandAnswers
Q.1) Consider the following statements regarding 'the Child and Adolescent Labour (Prohibition and
Regulation) Act of 1986':

1. It was amended in 2016.

2. The act also prohobits employment of child in non-hazardous family business.

Which of the following statements given above is/are correct?

a) 1 only

b) None is correct

c) Both are correct

d) 2 only

Answer: (a)

The Child and Adolescent Labour (Prohibition and Regulation) Act of 1986: The Act prohibits the
employment of children below the age of 14 years in hazardous occupations identified in a list by the law.
The list was expanded in 2006, and again in 2008. In 2016, the Act was amended (not in force as of 30
July 2016) to prohibit employment of child below 14 years in all occupation (except for helping in
non-hazardous family business and of child artists in the entertainment industry and sports). Further,
adolescents between 1418 years will not be allowed to work in hazardous industries and processes.

Q.2) The International Fund for Agricultural Development (IFAD) is a specialised agency of the

a) International Monetary Fund

b) World Bank

c) United Organisation

PrelimsDailyArchive-Clickhere|TargetMainsArchive-Clickhere

ADailyInitiativefromCivilsdaily|DownloadFREEApp|JointhePrelimsModule

d) Economic Cooperation Organization Trade and Development Bank (ETDB)

Answer: (c)

The International Fund for Agricultural Development (IFAD) is an international financial institution and a
specialised agency of the United Nations dedicated to eradicating poverty and hunger in rural areas of
developing countries. It was established as an international financial institution in 1977 as one of the
major outcomes of the 1974 World Food Conference.

Q.3) Which of the following is/are not (a)Indian Credit Agency?

1. ONICRA

2. S&P

3. SMERA

4. ERA

a) 2 only

b) 2 and 4 only

c) 1 and 4 only

d) 2 and 3

Answer: (b)

S&P is of US and ERA is of Europe.

Q.4) What is 'Protectionism' in Economics?

a) It is a method of avoiding unlawful use of Intellectual Property Rights

b) It is the economic policy of restraining trade between states (countries)

c) It is an economic policy to encourage Competitive federalism

d) None of the above

Answer: (b)

In economics, protectionism is the economic policy of restraining trade between states (countries) through
methods such as tariffs on imported goods, restrictive quotas, and a variety of other government

PrelimsDailyArchive-Clickhere|TargetMainsArchive-Clickhere

ADailyInitiativefromCivilsdaily|DownloadFREEApp|JointhePrelimsModule

regulations. Protectionist policies, in theory, protect the producers, businesses, and workers of the
import-competing sector in a country from foreign competitors.

Q.5) A nation's ranking on the ease of doing business index is based on the average of 10 subindices.
Which of the following is not one of those subindices?

a) Starting a business

b) Dealing with construction permits

c) Trading across borders

d) All of the above

Answer: (d)

Subindices: Starting a business, Dealing with construction permits, Getting electricity, Registering
property, Getting credit, Protecting investors, Paying taxes, Trading across borders, Enforcing contracts,
Resolving insolvency.

Q.6) Consider the following pairs :

Tribe State

1. Limboo (Limbu) : Sikkim

2. Karbi : Himachal Pradesh

3. Dongaria : Odisha

4. Bonda : Tamil Nadu

Which of the above pairs are correctly matched?

a) 1 and 3 only

b) 2 and 4 only

c) 1, 3 and 4 only

d) 1, 2, 3 and 4

Answer: (a)

Limbu tribe is from Sikkim and Dongaria Kondh tribe is from Odisha ( Karbi is from Assam and Bonda is
in Odisha).

PrelimsDailyArchive-Clickhere|TargetMainsArchive-Clickhere

ADailyInitiativefromCivilsdaily|DownloadFREEApp|JointhePrelimsModule

Q.7) Consider the following liquid assets:

1.Demand deposits with the banks

2.Time deposits with the banks

3.Savings deposits with the banks

4.Currency

The correct sequence of these decreasing order of Liquidity is

a) 1432

b) 4321

c) 2314

d) 4132

Answer: (d)

Liquidity of currency is highest. Next will come demand liabilities, as they have to be cleared when
demanded. Next savings and least liquidity will be of time deposits

Q.8) In the context of Indian economy,Open Market Operations' refers to

a) borrowing by scheduled banks from the RBI

b) lending by commercial banks to industry and trade

c) purchase and sale of government securities by the RBI

d) None of the above

Answer: (c)

The repo and reverse repo rates are used when RBI purchase or sale the government securities. This
route is known as Open Market Operations.

PrelimsDailyArchive-Clickhere|TargetMainsArchive-Clickhere

ADailyInitiativefromCivilsdaily|DownloadFREEApp|JointhePrelimsModule

PRELIMSDAILY
8Questions+1Tikdam/Titbitsinexplanations

----------------------------------------------------------------

22June2017QuestionsandAnswers
Q.1) Consider the following statements regarding 'Solar Cells':

1. Solar cells converts solar energy into a usable amount of direct current (DC) electricity.

2. Cadmium telluride is used for making Solar Cells.

Which of the following statements given above is/are correct?

a) None is correct

b) 2 only

c) 1 only

d) Both are correct

Answer: (d)

Photons in sunlight hit the solar panel and are absorbed by semiconducting materials, such as silicon. An
array of solar cells converts solar energy into a usable amount of direct current (DC) electricity. An
inverter can convert the power to alternating current (AC).

Q.2) Which of the following Act created 'Board of Control' to manage the political affairs during the
Company Rule(1773-1858)?

a) The Charter Act of 1853

b) The Pitts India Act of 1784

c) The Charter Act of 1833

d) 'Board of Control' was not created during Company Rule

PrelimsDailyArchive-Clickhere|TargetMainsArchive-Clickhere

ADailyInitiativefromCivilsdaily|DownloadFREEApp|JointhePrelimsModule

Answer: (b)

The Pitt's India Act of 1784 allowed the Court of Directors to manage the commercial affairs but created a
new body called Board of Control to manage the political affairs. Thus, it established a system of double
government. It empowered the Board of Control to supervise and direct all operations of the civil and
military government or revenues of the British possessions in India.

Q.3) Consider the following statements:

1. The AIP module is being developed by Indian Space Research Organization.

2. The module enables conventional submarines to stay remain underwater for a longer duration.

Which of the following statements are correct?

a) 1 only

b) 2 only

c) Both 1 and 2

d) Neither 1 nor 2

Answer: (b)

The AIP module is being developed by Defence Research and Development Organisation (DRDO). The
module enables conventional submarines to stay remain underwater for a longer duration.

Source: http://www.civilsdaily.com/scorpene-submarines-to-join-navy-without-aip-modules/

Q.4) Consider the following statements about land revenue settlements during British India:

1. In permanent settlement, land revenue was fixed.

2. In Mahalwari settlement, land revenue was revised periodically.

3. In ryotwari settlement, land revenue was neither fixed nor revised periodically.

Which of the statements given above is/are correct?

a) 1 only

PrelimsDailyArchive-Clickhere|TargetMainsArchive-Clickhere

ADailyInitiativefromCivilsdaily|DownloadFREEApp|JointhePrelimsModule

b) 3 only

c) 1 and 2 only

d) 1,2 and 3

Answer: (c)

Q.5) Consider the following regarding Cellulose:

1. Cellulose is the most abundant organic polymer on Earth

2. Cellulose fibers can be obtained from the bark, wood or leaves of plants

Which of the above is/are correct?

a) 1 only

b) 2 only

c) Both are correct

d) Neither 1 nor 2

Answer: (c)

Cellulose is an important structural component of the primary cell wall of green plants, many forms of
algae and the oomycetes. Cellulose is the most abundant organic polymer on Earth. Cellulose fibers are
fibers made with ether or esters of cellulose, which can be obtained from the bark, wood or leaves of
plants, or from a plant-based material.

Q.6) Priority Sector Lending by banks in India constitutes the lending to(CSE: 2017)

a) agriculture

b) micro and small enterprises

c) weaker sections

d) All of the above

Answer: (d)

PrelimsDailyArchive-Clickhere|TargetMainsArchive-Clickhere

ADailyInitiativefromCivilsdaily|DownloadFREEApp|JointhePrelimsModule

Banks have to lend minimum of 40% to priority sector which includes all of the given sectors

Q.7) Which one among the following industries is the maximum consumer of water in India?(CSE: 2017)

a) Engineering

b) Paper and pulp

c) Textiles

d) Thermal power

Answer: (d)

Of the total water use by the industry, thermal power plants are the biggest users of water and account for
88% of the total industrial water use. They are followed by engineering (5.05%) pulp

Q.8) To obtain full benefits of demographic dividend, what should India do?(CSE: 2017)

a) Promoting skill development

b) Introducing more social security schemes

c) Reducing infant mortality rate

d) Privatization of higher education

Answer: (a)

b option will not provide any benefit of demographic dividend. Social security schemes can be used for
providing security to non-working population like old-age pension etc.c option is one of the Millenium
Development Goal, but it does not provide any benefit of demographic dividendd option can infact have
negative effect as it will make education costlier thus making it out of reach of a large section of eligible
population

PrelimsDailyArchive-Clickhere|TargetMainsArchive-Clickhere

ADailyInitiativefromCivilsdaily|DownloadFREEApp|JointhePrelimsModule

PRELIMSDAILY
8Questions+1Tikdam/Titbitsinexplanations

----------------------------------------------------------------

23June2017QuestionsandAnswers
Q.1) Which sector/user among the following has the lowest share of Electricity Consumption in India?

a) Residential consumption

b) Industrial

c) Agriculture

d) Share that is lost during Transmission & Distribution

Answer: (c)

2015-2016 Data

Transmission & Distribution losses: 22.77%

Residential consumption: 22.97%

Industrial consumption: 44.20%

Agriculture consumption: 18.17%

Q.2) The National Company Law Tribunal (NCLT) is a quasi-judicial body in India that adjudicates issues
relating to companies in India. It was establsihed under/by the

a) SEBI

b) Reserve Bank of India

c) insolvency and bankruptcy code 2016

d) Companies Act 2013

Answer: (d)

PrelimsDailyArchive-Clickhere|TargetMainsArchive-Clickhere

ADailyInitiativefromCivilsdaily|DownloadFREEApp|JointhePrelimsModule

The National Company Law Tribunal (NCLT) is a quasi-judicial body in India that adjudicates issues
relating to companies in India. The NCLT was established under the Companies Act 2013 and was
constituted on 1 June 2016. The NCLT has eleven benches, two at New Delhi (one being the principal
bench) and one each at Ahmedabad, Allahabad, Bengaluru, Chandigarh, Chennai, Guwahati, Hyderabad,
Kolkata and Mumbai. Justice M.M. Kumar, a retired judge of the Punjab and Haryana High Court has
been appointed as President of the NCLT. The NCLT Bench at Bangalore began functioning on 18 July
2016.

Q.3) Which Act during the Company Rule made the Governor-General of Bengal as the
Governor-General of India and vested in him all civil and military powers?

a) Pitts India Act of 1784

b) Charter Act of 1833

c) Charter Act of 1853

d) Regulating Act

Answer: (b)

Charter Act of 1833 made the Governor-General of Bengal as the Governor-General of India and vested
in him all civil and military powers. Thus, the act created, for the first time, a Government of India having
authority over the entire territorial area possessed by the British in India. Lord William Bentick was the first
governor-general of India. It deprived the governor of Bombay and Madras of their legislative powers. The
Governor-General of India was given exclusive legislative powers for the entire British India. The laws
made under the previous acts were called as Regulations while laws made under this act were called as
Acts.

Q.4) With reference to Balance of Payments, which of the following constitutes/constitute the Current
Account?

1. Balance of trade

2. Foreign assets

3. Balance of invisibles

4. Special Drawing Rights

Select the correct answer using the code given below

a) 1 only

PrelimsDailyArchive-Clickhere|TargetMainsArchive-Clickhere

ADailyInitiativefromCivilsdaily|DownloadFREEApp|JointhePrelimsModule

b) 2 and 3

c) 1 and 3

d) 1, 2 and 4

Answer: (c)

Current account includes balance of trade (exports-imports), net remittances, Trade in services. SDRs
count as foreign assets.

Q.5) Which of the following constitute Capital Account?

1. Foreign Loans

2. Foreign Direct Investment

3. Private Remittances

4. Portfolio Investment

Select the correct answer using the codes given below:

a) 1, 2 and 3

b) 1, 2 and 4

c) 2, 3 and 4

d) 1, 3 and 4

Answer: (b)

Private Remittances forms a part of current account and not capital account. Try remembering this by fact
that other three when enter economy, they will stay for some time and not be taken out instantly. But a
crisis in a country where many indian nationals work can lead to reduction in remittances i.e. it is current
in nature.

Q.6) In the context of cultural history of India, a pose in dance and dramatics called 'Tribhanga' has been
a favourite of Indian artists from ancient times till today. Which one of the following statements best
describes this pose?

a) One leg is bent and the body is slightly but oppositely curved at waist and neck

b) Facial expressions, hand gestures and makeup are combined to symbolize certain epic or historic
characters

PrelimsDailyArchive-Clickhere|TargetMainsArchive-Clickhere

ADailyInitiativefromCivilsdaily|DownloadFREEApp|JointhePrelimsModule

c) Movements of body, face and hands are used to express oneself or to tell a story

d) A little smile, slightly curved waist and certain hand gestures are emphasized to express the feelings of
love or eroticism

Answer: (a)

Tribhanga, literally meaning three parts break, consists of three bends in the body; at the neck, waist and
knee, hence the body is oppositely curved at waist and neck which gives it a gentle 'S' shape

Q.7) Annie Besant was

1. responsible for starting the Home Rule Movement

2. the founder of the Theosophical Society

3. once the President of the Indian National Congress Select the

correct statement/statements using the codes given below.

a) 1 only

b) 2 and 3 only

c) 1 and 3 only

d) 1, 2 and 3

Answer: (c)

Annie Besant was the 1st women President of INC in 1917 Calcutta Session. Madam Blavatsky and
Henry Steel Olcott founded Theosophical Society.

Q.8) The Ilbert Bill controversy was related to the

a) imposition restrictions the Indians of certain to carry arms by Indians

b) imposition of restrictions on newspapers and magazines published in Indian languages

c) removal of disqualifications imposed on the Indian magistrates with regard to the trial of the Europeans

d) removal of a duty on imported cotton cloth

Answer: (c)

PrelimsDailyArchive-Clickhere|TargetMainsArchive-Clickhere

ADailyInitiativefromCivilsdaily|DownloadFREEApp|JointhePrelimsModule

In AD 1883, Lord Ripons law member, Sir Ilbert, introduced a bill which came to be called the Ilbert Bill. It
allowed Indian judges to try the cases involving the Europeans.

PrelimsDailyArchive-Clickhere|TargetMainsArchive-Clickhere

ADailyInitiativefromCivilsdaily|DownloadFREEApp|JointhePrelimsModule

PRELIMSDAILY
8Questions+1Tikdam/Titbitsinexplanations

----------------------------------------------------------------

24June2017QuestionsandAnswers
Q.1) Which of the following countries are not part of the American Continent?

1. Belize

2. Haiti

3. Gabon

4. Lesotho

5. Dominica

Select the correct option using the codes given below.

a) 1, 2 and 5 only

b) 3 and 4 only

c) 1, 2 and 3 only

d) 4 and 5 only

Answer: (b)

Gabon and Lesotho are from African Continent.

Q.2) In which of the following states the Prevention of Corruption Act, 1988 is not applicable?

a) Nagaland and Mizoram

b) Nagaland and Manipur

c) Arunachal Pradesh and Megahlya

d) None of the Above

PrelimsDailyArchive-Clickhere|TargetMainsArchive-Clickhere

ADailyInitiativefromCivilsdaily|DownloadFREEApp|JointhePrelimsModule

Answer: (d)

The Prevention of Corruption Act, 1988 is not applicable to Jammu and Kashmir.

Q.3) Which of the following act during the Company Rule prohibited the servants of the Company from
engaging in any private trade?

a) Regulating Act of 1773

b) Pitts India Act of 1784

c) Charter Act of 1833

d) Charter Act of 1853

Answer: (a)

Regulating Act of 1773 prohibited the servants of the Company from engaging in any private trade or
accepting presents or bribes from the natives.

Q.4) Consider the following statements:

1. It was established by the enactment of the Tea Act in 1953 with its headquarters in Kolkata

2. It is a state agency of the Government of Kolkata

3. It is established to promote the cultivation, processing, and domestic trade as well as export of tea
from India

Which of the following are correct?

a) 1 only

b) 1,2 and 3

c) 1 and 3 only

d) 2 only

Answer: (c)

It is a state agency of the Government of India. It is established to promote the cultivation, processing,
and domestic trade as well as export of tea from India. It was established by the enactment of the Tea Act
in 1953 with its headquarters in Kolkata. The Tea Board India is responsible for the assignment of
certification numbers to exports of certain tea merchants.

PrelimsDailyArchive-Clickhere|TargetMainsArchive-Clickhere

ADailyInitiativefromCivilsdaily|DownloadFREEApp|JointhePrelimsModule

Q.5) Consider the following statements regarding Agriculture Insurance Company of India Limited (AIC):

1. Agricultural loans by banks in India are compulsorily insured by the Agricultural Insurance Company of
India

2. AIC also transacts other insurance businesses directly or indirectly concerning agriculture and its allied
activities

Which of the above is/are correct?

a) 1 only

b) 2 only

c) Both 1 and 2

d) Neither 1 nor 2

Answer: (c)

Agricultural loans by banks in India are compulsorily insured by the Agricultural Insurance Company of
India (AIC). Its liabilities are back-stopped by the Centre through budgetary support. AIC has taken over
the implementation of National Agricultural Insurance Scheme (NAIS). In addition, AIC also transacts
other insurance businesses directly or indirectly concerning agriculture and its allied activities.

Q.6) A rise in general level of prices may be caused by(CSE: 2013)

1. an increase in the money supply

2. a decrease in the aggregate level of output

3. an increase in the effective demand

Select the correct answer using the codes given below.

a) 1 only

b) 1 and 2 only

c) 2 and 3 only

d) 1, 2 and 3

Answer: (d)

PrelimsDailyArchive-Clickhere|TargetMainsArchive-Clickhere

ADailyInitiativefromCivilsdaily|DownloadFREEApp|JointhePrelimsModule

It a direct application based question. 1st statement is correct as increase in money supply will cause
inflation. 2nd and 3rd statement are obviously correct as they represent supply and demand side of
products

Q.7) Which one of the following groups of items is included in India's foreignexchange reserves?(CSE:
2013)

a) Foreigncurrency assets, Special Drawing Rights (SDRs) and loans from foreign countries

b) Foreigncurrency assets, gold oldings of the RBI and SDRs

c) Foreigncurrency assets, loans from the World Bank and SDRs

d) Foreigncurrency assets, gold holdings of the RBI and loans from the World Bank

Answer: (b)

India's foreign exchange reserves compriseforeign currency assets (FCA), gold, special drawing rights
(SDRs) and reserve tranche position (RTP) in the International Monetary Fund (IMF).

Q.8) Which one of the following is likely to be the most inflationary in its effect?(CSE: 2013)

a) Repayment of public debt

b) Borrowing from the public to finance a budget deficit

c) Borrowing from banks to finance a budget deficit

d) Creating new money to finance a budget deficit

Answer: (d)

b and c option will lead to a decrease in the money supply in market. Between a and d, d will have more
inflationary effect than a, as it will lead to an increase in total money supply in the market.

PrelimsDailyArchive-Clickhere|TargetMainsArchive-Clickhere

ADailyInitiativefromCivilsdaily|DownloadFREEApp|JointhePrelimsModule

PRELIMSDAILY
8Questions+1Tikdam/Titbitsinexplanations

----------------------------------------------------------------

26June2017QuestionsandAnswers
Q.1) Consider the following statements regarding the ' City Liveability Index' which was recently seen in
the news:

1. It will rank all districts on the basis of the quality of life they offer.

2. It was launched by the Union Government in collaboration with the UN-Habitat (United Nations Human
Settlements Programme)

Which of the following statements given above is/are correct?

a) Neither 1 nor 2

b) 2 only

c) Both are correct

d) 1 only

Answer: (a)

The Union government has recently launched a liveability index(City Liveability Index) which will rank the
countrys 116 major cities on the basis of the quality of life they offer. It will cover cities with population
above one million, including the capital cities. It will enable the cities know where they stand in terms of
the quality of life and the interventions required to improve it.

Q.2) Which of the following statements given below is/are correct regarding the Central Statistics Office
(CSO)?

1. It is responsible for the compilation of National Accounts Statistics (NAS).

2. The CSO was set up under the Government of India Act 1935.

Select the correct option using the codes given below.

PrelimsDailyArchive-Clickhere|TargetMainsArchive-Clickhere

ADailyInitiativefromCivilsdaily|DownloadFREEApp|JointhePrelimsModule

a) 2 only

b) Neither 1 nor 2

c) 1 only

d) Both are correct

Answer: (c)

The CSO is headed by the Director-General who is assisted by Five additional Director-Generals and four
Deputy Director-Generals, six Joint Directors,seven special task officers,thirty deputy directors, 48
assistant directors and other supporting staff. The CSO is located in Delhi. The CSO was set up in the
cabinet secretariat on 2 May 1951.

Q.3) Which of the following Constitutional Amendment is known as Mini-Constitution due to the important
and large number of changes made by it in various parts of the Constitution?

a) 73rd Amendment Act

b) 44th Amendment Act

c) 7th Amendment Act

d) 42nd Amendment Act

Answer: (d)

The 42nd Amendment Act (1976) is known as Mini- Constitution due to the important and large number
of changes made by it in various parts of the Constitution. However, in the Kesavananda Bharati case1
(1973), the Supreme Court ruled that the constituent power of Parliament under Article 368 does not
enable it to alter the basic structure of the Constitution.

Q.4) In case the market price for the commodity falls below the announced minimum price due to bumper
production and glut in the market:

a) government does not procure the farmer's produce anymore

b) only state governments do the procurement

c) government agencies purchase the entire quantity offered by the farmers at the announced minimum
price

d) no mechanism exists for any such condition

PrelimsDailyArchive-Clickhere|TargetMainsArchive-Clickhere

ADailyInitiativefromCivilsdaily|DownloadFREEApp|JointhePrelimsModule

Answer: (c)

MSP is price fixed by Government of India to protect the producer - farmers - against excessive fall in
price during bumper production years. The minimum support prices are a guarantee price for their
produce from the Government.

In case the market price for the commodity falls below the announced minimum price due to bumper
production and glut in the market, government agencies purchase the entire quantity offered by the
farmers at the announced minimum price.

Q.5) Consider the following statements regarding banking ombudsman:

1. Banking Ombudsman is a quasi judicial authority functioning under India's Banking Ombudsman
Scheme

2. Only Scheduled Commercial Banks are covered under this scheme.

Which of the above is/are correct?

a) 1 only

b) 2 only

c) Both 1 and 2

d) Neither 1 nor 2

Answer: (a)

The Banking Ombudsman Scheme is introduced under Section 35 A of the Banking Regulation Act, 1949
by RBI with effect from 1995.

All Scheduled Commercial Banks, Regional Rural Banks and Scheduled Primary Co-operative Banks are
covered under the Scheme.

Q.6) Supply of money remammg the same when there is an increase in demand for money, there will be
(CSE: 2013)

a) a fall in the level of prices

b) an increase in the rate of interest

c) a decrease in the rate of interest

d) an increase in the level of income and employment

PrelimsDailyArchive-Clickhere|TargetMainsArchive-Clickhere

ADailyInitiativefromCivilsdaily|DownloadFREEApp|JointhePrelimsModule

Answer: (b)

Direct application based question. If the demand increase when the supply is same, it will lead to an
increase in prices or in this case rate of interest to be charged.

Q.7) Fruits stored in a cold chamber exhibit longer storage life because (CSE: 2013)

a) exposure to sunlight is prevented

b) concentration dioxide in the is increased of carbon environment

c) rate of respiration is decreased

d) there is an increase in humidity

Answer: (c)

Respiration can be controlled by refrigerated storage.

Q.8) Consider the following fauna of India:(CSE: 2013)

1.Gharial

2.Leatherback turtle

3.Swamp deer

Which of the above is/are endangered?

a) 1 and 2 only

b) 3 only

c) 1, 2 and 3

d) None

Answer: (c)

All three are endangered in india and as there is no specific list(IUCN etc.) is given so option C is correct.

PrelimsDailyArchive-Clickhere|TargetMainsArchive-Clickhere

ADailyInitiativefromCivilsdaily|DownloadFREEApp|JointhePrelimsModule

PRELIMSDAILY
8Questions+1Tikdam/Titbitsinexplanations

----------------------------------------------------------------

27June2017QuestionsandAnswers
Q.1) Which of the following Fiber varieties, produced globally, are not vegetable fibers?

1. Coir

2. Cotton

3. Lenin

4. Jute

5. Hemp

Select the correct option using the codes given below.

a) All are vegetable fibers

b) 1, 2, 3 and 5 only

c) 1 and 3 only

d) 4 and 5 only

Answer: (a)

Types of fibre

Animal fibers- Angora , pashmina ,silk.

Vegetable fibers -coir , cotton ,Linen, jute, Hemp.

Mineral fibers asbestos.

The synthetic fibers are further classied into Celloluse , mineral and polymer. ...

Cellulose fibres- Art silk , Rayon.

PrelimsDailyArchive-Clickhere|TargetMainsArchive-Clickhere

ADailyInitiativefromCivilsdaily|DownloadFREEApp|JointhePrelimsModule

Mineral fibres- glass , basalt , metallic.

Q.2) Consider the following statements regarding the Morgan Stanley Capital International(MSCI):

1. It is a British provider of equity, fixed income, hedge fund stock market indexes, and equity portfolio
analysis tools.

2. It was formed just after the Great Depression of 1930.

Which of the following statements given above is/are correct?

a) 2 only

b) Neither 1 nor 2

c) Both are correct

d) 1 only

Answer: (b)

MSCI Inc. (formerly Morgan Stanley Capital International and MSCI Barra), is an American provider of
equity, fixed income, hedge fund stock market indexes, and equity portfolio analysis tools. It publishes the
MSCI BRIC, MSCI World and MSCI EAFE Indexes. The company is currently headquartered at 7 World
Trade Center in Manhattan, New York City, U.S.

It was founded in 1969.

Q.3) Totalisation agreement, sometimes seen in international news, is related to

a) Terrorism

b) Civil Nuclear Deals

c) Taxes

d) Intellectual property

Answer: (c)

Totalisation agreement: When employee from one country works in another and ends up paying taxes in
both the countries. Totalization helps by eliminating such dual Social Security taxation. It also helps in
filling the gaps in benefit of the protection for the workers who divided their career between two countries
by totalisation of the period of contribution in the host country for determining the eligibility for benefits.

PrelimsDailyArchive-Clickhere|TargetMainsArchive-Clickhere

ADailyInitiativefromCivilsdaily|DownloadFREEApp|JointhePrelimsModule

Q.4) Consider the following statement regarding the ' Gross Enrolment Ratio (GER)' which was recently
seen in news:

1. In India, GER is calculated for the age group of 18 to 23 years.

2. GER is not recognized by the UNESCO.

Which of the following statements given above is/are correct?

a) 2 only

b) Both are correct

c) Neither 1 nor 2

d) 1 only

Answer: (d)

Gross Enrollment Ratio (GER) or Gross Enrollment Index (GEI) is a statistical measure used in the
education sector and by the UN in its Education Index to determine the number of students enrolled in
school at several different grade levels (like elementary, middle school and high school), and use it to
show the ratio of the number of students who live in that country to those who qualify for the particular
grade level. The United Nations Educational, Scientific and Cultural Organization(UNESCO), describes
Gross Enrolment Ratio as the total enrolment within a country in a specific level of education, regardless
of age, expressed as a percentage of the population in the official age group corresponding to this level of
education.

Q.5) Keshavnanda Bharti Vs Union of India case of 1973 is related to which Article of the Indian
Constitution?

a) Article 343

b) Article 395

c) Article 368

d) Article 356

Answer: (c)

in the Kesavananda Bharati case1 (1973), the Supreme Court ruled that the constituent power of
Parliament under Article 368 does not enable it to alter the basic structure of the Constitution.

PrelimsDailyArchive-Clickhere|TargetMainsArchive-Clickhere

ADailyInitiativefromCivilsdaily|DownloadFREEApp|JointhePrelimsModule

Q.6) Ball bearings are used in bicycles, cars, etc., because(CSE: 2013)

a) the actual between the is increased area of contact wheel and axle

b) the effective between the is increased area of contact wheel and axle

c) the effective between the is reduced area of contact wheel and axle

d) None of the above statements is correct

Answer: (c)

The roundness of the bearings creates less contact between the bearing that is trying to move objects
and the object being moved. A reduction in contact area means less friction.

Q.7) Consider the following phenomena:(CSE: 2013)

1. Size of the sun at dusk

2. Colour of the sun at dawn

3. Moon being visible at dawn

4. Twinkle of stars in the sky

5. Polestar beingvisible in the sky

Which of the above are optical illusions?

a) 1, 2 and 3

b) 3, 4 and 5

c) 1, 2 and 4

d) 2, 3 and 5

Answer: (c)

Q.8) Rainbow is produced when sunlight falls on drops of rain.(CSE: 2013)

Which of the following physical phenomena are responsible for this?

1. Dispersion

2. Refraction

PrelimsDailyArchive-Clickhere|TargetMainsArchive-Clickhere

ADailyInitiativefromCivilsdaily|DownloadFREEApp|JointhePrelimsModule

3. Internal reflection

Select the correct answer using the codes given below.

a) 1 and 2 only

b) 2 and 3 only

c) 1 and 3 only

d) 1, 2 and 3

Answer: (d)

Each individual droplet of water acts as a tiny prism that both disperses the light and reflects it back to our
eye.Option:-2Since water is more dense than air, light is refracted as it enters the drop.Option:-3 Some of
the light will reflect off the back of the drop if the angle is larger than the critical angle (48 for water).

PrelimsDailyArchive-Clickhere|TargetMainsArchive-Clickhere

ADailyInitiativefromCivilsdaily|DownloadFREEApp|JointhePrelimsModule

PRELIMSDAILY
8Questions+1Tikdam/Titbitsinexplanations

----------------------------------------------------------------

28June2017QuestionsandAnswers
Q.1) Which of the following statement is not correct with reference to Exclusive Economic Zones?

a) Coastal states of any country have no sovereign rights over the natural resources within this zone.

b) It is a zone which extends for 200 nautical miles beyond the baseline around the coast of any country.

c) USA has the largest Exclusive Zones in the world.

d) None of the above

Answer: (a)

Coastal states of any country have sovereign rights over the natural resources within this zone.

Q.2) Consider the following statements regarding GST:

1. The country will switch to the GST regime from July 1, 2017 including Jammu and Kashmir

2. Under GST, it is mandatory to pass on the benefit due to reduction in rate of tax or from input tax credit
to the consumer

Which of the above is/are correct?

a) 1 only

b) 2 only

c) Both 1 and 2

d) Neither 1 nor 2

Answer: (b)

PrelimsDailyArchive-Clickhere|TargetMainsArchive-Clickhere

ADailyInitiativefromCivilsdaily|DownloadFREEApp|JointhePrelimsModule

Barring the State of Jammu and Kashmir, the country will switch to the GST regime from July 1, 2017.
Clause 171 has been inserted in the GST bill which provides that it is mandatory to pass on the benefit
due to reduction in rate of tax or from input tax credit to the consumer by way of commensurate reduction
in prices.

Q.3) Consider the following statements regarding T Bills:

1. Treasury Bills are long term borrowing instruments of the Government of India

2. They are auctioned by Reserve Bank of India once in every 10 years

Which of the above is/are incorrect?

a) 1 only

b) 2 only

c) Both 1 and 2

d) Neither 1 nor 2

Answer: (c)

Treasury Bills are short term (up to one year) borrowing instruments of the Government of India which
enable investors to park their short term surplus funds while reducing their market risk. They are
auctioned by Reserve Bank of India at regular intervals and issued at a discount to face value.

Q.4) Consider the following Mountain Passes of Himalayan region:

1. Nathu La

2. Jalep La

3. Donkha La

4. Goecha La

5. Cho La

Which of the Mountain Passes given above are from Sikkim?

a) 2, 3 and 5 only

b) All of the above

c) 1, 3 and 4 only

PrelimsDailyArchive-Clickhere|TargetMainsArchive-Clickhere

ADailyInitiativefromCivilsdaily|DownloadFREEApp|JointhePrelimsModule

d) 1, 2 and 5 only

Answer: (b)

The mountain chains which run southwards from the main Himalayan range form the natural boundaries
of Sikkim. Located between these towering mountain ranges are passes like Nathu-la, Jelep-la, Cho-la
and many others which were at one time important corridors of passage between Sikkim and Tibet.

Q.5) Consider the following statements regarding the Special Purpose Vehicle(SPV) which is often seen
in the news:

1. It is a subsidiary company with an asset/liability structure.

2. It is a legal entity.

Which of the following statements given above is/are correct?

a) 2 only

b) 1 only

c) None is correct

d) Both are correct

Answer: (d)

A special purpose vehicle/entity (SPV/SPE) is a subsidiary company with an asset/liability structure and
legal status that makes its obligations secure even if the parent company goes bankrupt. An SPV/SPE is
also a subsidiary corporation designed to serve as a counterparty for swaps and other credit sensitive
derivative instruments.

Q.6) Many transplanted seedlings do not grow because(CSE: 2013)

a) the new soil does not contain favourable minerals

b) most of the root hairs grip the new soil too hard

c) most of the root hairs are lost during transplantation

d) leaves get damaged during transplantation

Answer: (c)

Newly transplanted trees or shrubs experience some degree of transplant shock. This is simply the plant
reacting to having been moved. It is generally caused by damage to the roots of the plant during the

PrelimsDailyArchive-Clickhere|TargetMainsArchive-Clickhere

ADailyInitiativefromCivilsdaily|DownloadFREEApp|JointhePrelimsModule

transplant. While the thickest roots are nearest to the root ball, the most important roots, those that
actually 'do the work', are located the farthest away from the plant. These tiny roots are covered by even
tinier hairs which absorb most of the water that is eventually carried to the top-growth of the plant.During
a transplant, these fine roots are usually destroyed either by being cut to keep the root ball size down, by
drying out (some accounts say it only takes 3-4 minutes of exposure to air for these to die), or simply by
being jarred and jostled around in the move. As a result of the loss of these feeder roots, the plant is not
able to draw the amount of moisture it requires to thrive and grow. The resulting imbalance between the
moisture demands of the top-growth and the reduced ability of the roots to meet those demands
manifests as transplant shock.

Q.7) Economic growth in country X will necessarily have to occur if(CSE: 2013)

a) there is technical progress in the world economy

b) there is population growth in X

c) there is capital formation in X

d) the volume of trade grows in the world economy

Answer: (c)

a and d statement does not say anything about the country X, only the general world economy. In b, if
there is population growth without any avenues for the economy to absorb them, it will have a
deteoriarating effect. c is the most appropriate option, as whenever there is capital formation in country it
will lead to growth.

Q.8) Which of the following statements is / are correct?(CSE: 2013)

1. Viruses lack enzymes necessary for the generation of energy.

2. Viruses can be cultured in any synthetic medium.

3. Viruses are transmitted from one organism to another by biological vectors only.

Select the correct answer using the codes given below.

a) 1 only

b) 2 and 3 only

c) 1 and 3 only

d) 1, 2 and 3

PrelimsDailyArchive-Clickhere|TargetMainsArchive-Clickhere

ADailyInitiativefromCivilsdaily|DownloadFREEApp|JointhePrelimsModule

Answer: (a)

Option 1 is right because viruses do not have the chemical machinery needed to survive on their own.
Option 2 is wrong because they seek out host cells in which they can multiply. Viruses can't be grown on
synthetic media (e.g. agar [a type of geletin]) but they can be grown on cultures of bacteria. Bacteria
reproduce and live like any other living organism would, it takes in nutrients and reproduces using binary
fission (like mitosis).Viruses on the other hand are like little aliens. They really aren't 'living' since they
don't absorb nutrients or produce waste. Rather they hook onto another cell (e.g. a bacterium) and inject
the cell with its own viral genetic code. This code then causes the cell to produce the multiple copies of
the virus, essentially doing the reproduction for the virus at the cost of its own resources.The virus can't
reproduce itself without another cell to 'build' its copies. Therefore we can't grow a virus on a synthetic
media .Option 3 is also wrong because viruses enter the body from the environment or other individuals
from soil to water to air via nose, mouth, or any breaks in the skin and seek a cell to infect.

PrelimsDailyArchive-Clickhere|TargetMainsArchive-Clickhere

ADailyInitiativefromCivilsdaily|DownloadFREEApp|JointhePrelimsModule

PRELIMSDAILY
8Questions+1Tikdam/Titbitsinexplanations

----------------------------------------------------------------

29June2017QuestionsandAnswers
Q.1) Consider the term 'GeneXpert' device which was recently seen in the news:

1. The World Health Organization (WHO) is recommending the use of GeneXpert.

2. This device can be used for Cancer Diagnosis.

Which of the following statements given above is/are correct?

a) 2 only

b) None is correct

c) 1 only

d) Neither 1 nor 2

Answer: (c)

The World Health Organization (WHO) has released a new device 'GeneXpert' which can be used for
multi-disease testing. The WHO is recommending use of these state-of-the-art portable machines. These
devices are based on Molecular Technology. The GeneXpert can be used to diagnose TB and HIV
infections, and quantitatively measure HIV and hepatitis C viral loads.

Q.2) Which Government Organisation/Ministry has released the Draft National Energy Policy?

a) Ministry of Finance

b) Ministry of Power

c) The National Development Council (NDC)

d) NITI Aayog

Answer: (d)

PrelimsDailyArchive-Clickhere|TargetMainsArchive-Clickhere

ADailyInitiativefromCivilsdaily|DownloadFREEApp|JointhePrelimsModule

NITI Aayog has Released the Draft National Energy Policy. Draft of the policy aims to find a way to
achieve the energy security targets announced by the government. Niti Aayog has recommended that the
seven subsidiaries of Indias largest coal miner Coal India Ltd should be converted into independent
companies. And these subsidiaries be allowed to compete against each other in the open market.

Q.3) Which of the following organs of Human Body is made up of Cardiac Muscle?

1. Lungs

2. Heart

3. Abdomen

Select the correct option using the codes given below.

a) All of the above

b) 1 and 2 only

c) 2 only

d) 2 and 3 only

Answer: (c)

Cardiac muscle (heart muscle) is an involuntary, striated muscle that is found in the walls and histological
foundation of the heart, specifically the myocardium. Cardiac muscle is one of three major types of
muscle, the others being skeletal and smooth muscle.

Q.4) Consider the following statements regarding Airports Economic Regulatory Authority (AERA) :

1. AERA is a constitutional body established to determine tariff in respect of airport services provided at
all airports across the country

2. The AERA revises tariff every year based on recommendations of Cabinet secretary

Which of the above is/are correct?

a) 1 only

b) 2 only

PrelimsDailyArchive-Clickhere|TargetMainsArchive-Clickhere

ADailyInitiativefromCivilsdaily|DownloadFREEApp|JointhePrelimsModule

c) Both 1 and 2

d) Neither 1 nor 2

Answer: (d)

AERA is a statutory body established in 2009 to determine tariff in respect of airport services provided at
major airports across the country. The AERA revises tariff every five years, known as a control period,
considering the investment and expenditure incurred by the airport operator.

Q.5) Consider the following countries:

1. India

2. USA

3. Japan

Which of the above is/are part of Malabar Exercises?

a) 1 and 2

b) 2 and 3

c) 1 and 3

d) All of the above

Answer: (d)

Exercise Malabar is a trilateral naval exercise involving the United States, Japan and India as permanent
partners. MALABAR has been held alternatively off India and in the Western Pacific.

PrelimsDailyArchive-Clickhere|TargetMainsArchive-Clickhere

ADailyInitiativefromCivilsdaily|DownloadFREEApp|JointhePrelimsModule

Q.6) Which of the following leaf modifications occurs/occur in desert areas to inhibit water loss? (CSE:
2013)

1. Hard and waxy leaves

2. Tiny leaves or no leaves

3. Thorns instead of leaves

Select the correct answer using the codes given below.

a) 1 and 2 only

b) 2 only

c) 1 and 3 only

d) 1, 2 and 3

Answer: (d)

Plants have few or no leave and foilage is either vaxy hairy or needle shaped to prevent water loss.

Q.7) The known forces of nature can be divided into four classes, viz, gravity electromagnetism, weak
nuclear force and strong nuclear force. with reference to them, which one of the following statements is
not correct?(CSE: 2013)

a) Gravity is the strongest of the four

b) Electromagnetism act only on particles with an electric charge

c) Weak nuclear force causes radioactivity

d) Strong nuclear force holds protons and nutrons inside the nuclear of an atom.

Answer: (a)

Gravity, the weakest of the four forces, is about 10^-36 times the strength of the strong force. This
weakness is easily demonstrable - on a dry day, when we rub a comb across shirt to give it static
electricity and when we hold it over a piece of paper on a desk the piece of paper lifts off the desk. It
takes an entire planet to keep the paper on the desk, but this force is easily overcome with everyday
materials employing the electromagnetic force.

Q.8) The efforts to detect the existence of Higgs boson particle have become frequent news in the recent
past. What is /are the importance/importances of discovering this particle?(CSE: 2013)

PrelimsDailyArchive-Clickhere|TargetMainsArchive-Clickhere

ADailyInitiativefromCivilsdaily|DownloadFREEApp|JointhePrelimsModule

1. It will enable us to understand as to why elementary particles have mass.

2. It will enables us in the near future to develope the technology to transferring matter from one point to
another without traversing the physical space between them.

3. It will enable us to create better fuels for nuclear fission.

Select the correct answer using the codes given below:

a) 1 only

b) 2 and 3 only

c) 1 and 3 only

d) 1, 2 and 3

Answer: (a)

Option 1:-The Higgs boson has long been thought the key to resolving the mystery of the origin of mass.
The Higgs boson is associated with a field, called the Higgs field, theorized to pervade the universe. As
other particles travel though this field, they acquire mass much as swimmers moving through a pool get
wet, the thinking goes. Option 2 and 3 :-(Scientist are not sure about these discoveries yet so should be
considered as Wrong/Doubtful)How many new discoveries will Higgs Boson help to make for the world?
Higgs Boson is not a main item that will open the way for teleportation but it will help to understand about
mass, item and travel in time and space. In the close or far future, Higgs Boson may help us to create an
unbelievable energy source.

PrelimsDailyArchive-Clickhere|TargetMainsArchive-Clickhere

ADailyInitiativefromCivilsdaily|DownloadFREEApp|JointhePrelimsModule

PRELIMSDAILY
8Questions+1Tikdam/Titbitsinexplanations

----------------------------------------------------------------

30June2017QuestionsandAnswers
Q.1) Two main passes between India and China open up in Chumbi Valley, Sikkim. Which of the following
are those two passes?

1. Nathu La Pass

2. Jelep La Pass

3. Goecha La

Select the correct option using the codes given below.

a) 1 and 2 only

b) 2 and 3 only

c) 1 and 3 only

d) 1 only and the other Pass is not given the question.

Answer: (a)

Chumbi Valley is a valley in Tibet at the intersection of India (Sikkim), Bhutan and China (Tibet) in the
Himalayas. Two main passes between India and China open up here: the Nathu La Pass and Jelep La
Pass. Administratively, the valley is in Yadong County of the Tibetan Autonomous Region.

Q.2) Odisha government has recently developed a real time flood forecasting model on one of its River.
This Model is developed on the river

a) Bhargavi

b) Daya

c) Devi

PrelimsDailyArchive-Clickhere|TargetMainsArchive-Clickhere

ADailyInitiativefromCivilsdaily|DownloadFREEApp|JointhePrelimsModule

d) Mahanadi

Answer: (d)

It is developed for the entire Mahanadi river basin for better flood control. The model is supported by the
State Climate Change Innovation Programme.

Q.3) Ransomware is carried out using:

a) A Trojan

b) A Spam

c) Boot virus

d) Encrypted virus

Answer: (a)

Source: http://www.civilsdaily.com/held-to-ransomware/

Q.4) GSAT17 was launched by/using:

a) Indian Space Research organization

b) National Aeronautics Space Administration

c) European Ariane 5 Launch Vehicle

d) International Space Science Institute

Answer: (c)

GSAT-17 was launched using the European Ariane 5 Launch Vehicle from Kourou, French Guiana.

Source: http://www.civilsdaily.com/pib-know-about-gsat-17-communication-satellite/

Q.5) HELP, NELP and OALP are:

a) IMF subsidiaries in Asia, Africa and Europe

b) Government schemes for Women

c) Goods and service tax forms

d) Names of hydrocarbon policies

PrelimsDailyArchive-Clickhere|TargetMainsArchive-Clickhere

ADailyInitiativefromCivilsdaily|DownloadFREEApp|JointhePrelimsModule

Answer: (d)

Hydrocarbon Exploration and Licensing Policy (HELP), New Exploration Licensing Policy (NELP) and
Open Acreage Licensing Policy (OALP) are names of different policies of government for exploration &
production of Oil & gas in the country

Q.6) Mycorrhizal biotechnology has been used in rehabilitating degraded sites because mycorrhiza
enables the plants to(CSE: 2013)

1. resist drought and increase absorptive area

2. tolerate extremes of PH

3. Resist disease infestation

Select the correct answer using the codes given below:

a) 1 only

b) 2 and 3 only

c) 1 and 3 only

d) 1, 2 and 3

Answer: (d)

Q.7) Who among the following constitute the National Development Council?(CSE: 2013)

1. The Prime Minister

2. The Chairman, Finance Commission

3. Ministers of the Union Cabinet

4. Chief Ministers of the States

Select the correct answer using the codes given below:

a) 1, 2 and 3 only

b) 1, 3 and 4 only

c) 2 and 4 only

d) 1, 2, 3 and 4

PrelimsDailyArchive-Clickhere|TargetMainsArchive-Clickhere

ADailyInitiativefromCivilsdaily|DownloadFREEApp|JointhePrelimsModule

Answer: (b)

The National Development Council includes the Prime Minister, Union ministers, Chief ministers of all
states, administrative heads of the Union Territories and members of the planning commission.

Q.8) The national income of a country for a given period is equal to the(CSE: 2013)

a) total value of goods and services produced by the nationals

b) sum of total consumption and investement expenditure

c) sum of personal income of all individuals

d) money value of final goods and services produced

Answer: (d)

PrelimsDailyArchive-Clickhere|TargetMainsArchive-Clickhere

Potrebbero piacerti anche